ATI Activity, ATI Case Studies, PN Pharmacology Online Practice 2017 A&B Questions with Rationales

Réussis tes devoirs et examens dès maintenant avec Quizwiz!

Match the drug with the therapeutic use 1. Baclofen 2. Amphetamine/dextroamphetamine sulfate 3. Dantrolene 4. Fentanyl A. Malignant hyperthermia B. Muscle spasms C. Anesthesia or cancer pain management D. Attention-deficit/hyperactivity disorder

1. > B, 2. > D, 3. A, 4. > C Baclofen: Muscle spasms Amphetamine/dextroamphetamine sulfate: Attention-deficit/hyperactivity disorder Dantrolene: Malignant hyperthermia Fentanyl: Anesthesia or cancer pain management Dantrolene is peripherally acting muscle relaxant that, in its IV form, helps reverse malignant hypertension, which is a dangerous complication of general anesthesia. The amphetamine mixture that comprises the brand-name drug Adderall helps clients with attention-deficit/hyperactivity disorder focus and lengthen their attention span. Baclofen helps manage muscle spasms and rigidity. Fentanyl provides cancer pain relief and is also used in anesthesia. (The Neurological System Activity (Part 1))

Match the drug with the therapeutic use 1. Heparin 2. Factor VIII 3. Filgrastim 4. Folic acid A. Hemophilia A B. Chronic neutropenia C. Deep vein thrombosis D. Megaloblastic anemia

1. > C, 2. > A, 3. > B, 4. > D Heparin: Deep vein thrombosis Factor VIII: Hemophilia A Filgrastim: Chronic neutropenia Folic acid: Megaloblastic anemia Folic acid treats megaloblastic anemia due to malabsorption syndrome, alcohol abuse, or hepatic disease. Factor VIII treats hemophilia A on demand for bleeding episodes or prophylactically to prevent bleeding. Filgrastim treats severe chronic neutropenia and reduces the infection risk associated with bone marrow transplantation. Heparin reduces clotting for an evolving cerebrovascular accident, pulmonary embolism, and deep vein thrombosis. It prevents postoperative venous thrombosis and treats acute myocardial infarction and disseminated intravascular coagulation. (The Hematologic System Activity)

When talking with a 30-year-old woman who will receive radioactive iodine-131 (Iodotope) to treat Graves' disease, you should include which of the following instructions? (Select all that apply.) A. Report weight gain and edema B. Use effective contraception C. Allow 2 to 3 months for full effects D. Expect periodic blood sampling E. Obtain regular eye examinations

A, B, C, D A. Report weight gain and edema B. Use effective contraception C. Allow 2 to 3 months for full effects D. Expect periodic blood sampling Radioactive iodine-131, an antithyroid drug, can cause hypothyroidism. Instruct the client to report any indications of hypothyroidism, such as drowsiness, depression, weight gain, or edema. Radioactive iodine-131 is a (The Endocrine System Activity)

You should monitor a client throughout continuous heparin therapy for which of the following adverse reactions? A. Thrombocytopenia B. Hypotension C. Hypokalemia D. Deep vein thrombosis E. Fever

A, B, D, E A. Thrombocytopenia B. Hypotension D. Deep vein thrombosis E. Fever For a client on continuous heparin therapy you should monitor platelet count for thrombocytopenia and stop heparin for any sudden drop in platelets. Clients receiving heparin are at risk for bleeding, which can cause hypotension and tachycardia. Heparin-induced thrombocytopenia produces antibodies that cause thrombus formation, so you should watch for indications of a new blood clot. Heparin can cause hypersensitivity reaction, which manifests as fever, chills, and hives. It is essential to administer a test does to determine hypersensitivity. Heparin is more likely to cause hyperkalemia than hypokalemia. (The Hematologic System Case Study)

You are caring for a client who is receiving alteplase to dissolve a thrombus. Which of the following should you include in your care plan for this client? (Select all that apply.) A. Monitor temperature B. Reposition frequently C. Limit venipunctures D. Assess neurologic status E. Apply pressure to oozing sites

A, C, D, E A. Monitor temperature C. Limit venipunctures D. Assess neurologic status E. Apply pressure to oozing sites Alteplase, a thrombolytic, can cause bleeding. you should monitor venipuncture sites and open wounds for bleeding and apply pressure dressings. Avoid or minimize venipunctures. Limit the clients movement to prevent further bleeding or trauma. Monitor neurologic status such as the level of consciousness and pupils to assess for intracranial bleeding. Alteplase can also cause a fever. Monitor temperature and, for fever, administer acetaminophen. (The Hematologic System Activity)

Which of the following information in the clients history causes the health care provider to suspect the client has osteoporosis? (Select all that apply.) A. Menopause B. Calcium supplements C. Long-term corticosteroid use D. Weight E. Loss of height

A, C, E A. Menopause C. Long-term corticosteroid use E. Loss of height Menopause and long-term corticosteroid use are causes of osteoporosis due to increased bone resorption. Loss of height may be indication of a compression fracture in the spine due to diminished bone density. Osteoporosis is more common in women who are thin and have low body mass index (BMI). The calcium supplements are not an indication of osteoporosis. Calcium supplements can help prevent osteoporosis. (The Musculoskeletal System Case Study)

After the client is reassessed, the provider prescribes a stat parenteral dose of a glucocorticoid. Recognizing the adverse effects of glucocorticoids, you should monitor the client for which of the following? (Select all that apply.) A. Hyperglycemia B. Thrombophlebitis C. Dysrhythmias D. Infection E. Gastric bleeding

A, D, E A. Hyperglycemia D. Infection E. Gastric bleeding Glucocorticoids may cause glucose intolerance and hyperglycemia. It is essential to monitor glucose levels for clients receiving these drugs. They may also increase the risk for or mask an existing infection. They can also increase gastric acid secretion, causing peptic ulcer disease with possible gastric bleeding. These drugs are not known to cause thrombophlebitis or dysrhythmias. (The Respiratory System Case Study)

When instructing the client about alendronate, which of the following information should the health care provider include? (Select all that apply.) A. Take the drug on an empty stomach. B. Take the drug before bedtime. C. Take the drug with an antacid. D. Sit upright for 30 minutes following dosage. E. Take with a full glass of water.

A, D, E A. Take the drug on an empty stomach. D. Sit upright for 30 minutes following dosage. E. Take with a full glass of water. Alendronate should be taken in the morning on an empty stomach for optimal bioavailability. Alendronate should be taken first thing in the morning before breakfast. Clients should wait 30 minutes before taking calcium or an antacid that might reduce the absorption of alendronate. Following dosage, the client should sit or stand upright for at least 39 minutes to reduce the risk of esophagitis. Clients should wait 30 minutes before taking calcium or an antacid that may reduce absorption of alendronate. Alendronate should be taken with a full glass of water to reduce the risk of esophagitis. (The Musculoskeletal System Case Study)

A nurse is collecting data from a client who is receiving digoxin for treatment of heart failure. The nurse should identify which of the following as adverse effects of this medication? (Select all that apply.) A. Blurred vision B. Nausea C. Hyperactivity D. Increased appetite E. Dysrhythmia

A. Blurred vision: The nurse should identify visual changes such as blurred visions, halos, and yellow or green tinge to vision as adverse effects of digoxin. B. Nausea: The nurse should identify that nausea and vomiting are adverse effects of digoxin. C. Dysrhythmia: The nurse should identify that dysrhythmias are adverse effect of digoxin. Hyperactivity is incorrect: The nurse should identify that fatigue and weakness are adverse effects of digoxin. increased appetite is incorrect: The nurse should identify that anorexia is an adverse effect of digoxin.

A nurse is caring for a client who has tuberculosis and will begin taking isoniazid. Which of the following actions should the nurse take? A. Determine the client's daily alcohol intake. B. Tell the client to expect red-orange colored urine. C. Reinforce teaching on a low-calorie diet. D. Instruct the client to have yearly tuberculin skin test.

A. Determine the client's daily alcohol intake.: The nurse should instruct the client to reduce or avoid all use of alcohol because isoniazid can cause liver damage; therefore. It is important for the nurse to determine the clients daily alcohol intake. Tell the client to expect red-orange colored urine.: Isoniazid does not cause red-orange colored urine. Rifampin is a medication used to treat tuberculosis and causes the clients sweat, urine, saliva, and tears to appear red-orange in color. Reinforce teaching on a low-calorie diet.: The client is at risk for losing weight; therefore, the nurse should reinforce how the client can maintain his weight with good nutrition. Instruct the client to have yearly tuberculin skin test.: A client who has tuberculosis will always have a positive tuberculin skin test. Therefore, there is no reason for the client to have a yearly test.

A client who has rheumatoid arthritis is taking methotrexate. Which of the following reduces toxicity from this drug? A. Folic acid B. Magnesium sulfate C. Ferrous sulfate d. Niacin

A. Folic acid Folic acid may reduce the toxic effects of methotrexate. The client should take 5 milligrams or more of folic acid supplement per week. Magnesium sulfate helps relieve acute constipation. Ferrous sulfate corrects iron deficiency anemia. Vitamins B3 has many therapeutic uses, including the prevention and treatment of pellagra, a nutritional deficiency with many dermatologic manifestations. (The Musculoskeletal System Activity)

A nurse is instilling timolol eyedrops for a client who has glaucoma. Which of the following actions should the nurse take after instilling the eyedrops? A. Press the nasolacrimal duct. B. Apply pressure to the upper eyelid. C. Ask the client to blink her eyes several times. D. Tell the client to keep her eyes open for at least 15 seconds.

A. Press the nasolacrimal duct.: The nurse should press the clients nasolacrimal duct after instilling the eye drops to prevent the medication from absorbing into systemic circulation. Apply pressure to the upper eyelid.: The nurse should avoid applying pressure to the clients upper eyelid after instilling the eye drops because it may force the medication out of the conjunctival sac. Ask the client to blink her eyes several times.: The nurse should avoid asking the client to blink her eyes after instilling the eyedrops because it can prevent the medication from entering the conjunctival sac. Tell the client to keep her eyes open for at least 15 seconds.: The nurse should avoid asking the client to keep her eyes open after instilling the eyedrops because it can inhibit distribution of the medication.

A nurse is collecting data from a client who has been taking levodopa/carbidopa. Which of the following findings should indicate to the nurse that the medication is effective? A. The client is able to wash his face. B. The client experiences fewer seizures. C. The client reports decreased heartburn. D. The client is able to sleep through the night.

A. The client is able to wash his face.: Levodopa works by activating dopamine receptors, restoring nerve transmission for clients who have Parkinson's disease. Carbidopa enhances these effects by inhibiting the breakdown of levodopa in the intestine and periphery. These therapeutic effects assist the client with moving freely and resuming ADLs. The client experiences fewer seizures.: Levodopa/carbidopa does not have antiseizure effects. Abnormal involuntary movements are an adverse effect of levodopa and can be reduced by amantadine administration. The client reports decreased heartburn.: Levodopa/carbidopa does not treat heartburn. Nausea and vomiting are adverse effects of levodopa. The client is able to sleep through the night.: Levodopa/carbidopa does not facilitate sleep. Vivid dreams and hallucinations are adverse effects of levodopa.

A nurse is caring for a client who has a new prescription for sumatriptan. the nurse notes that the client takes fluoxetine. The nurse should notify the provider that the combination of these medication s will place the client at risk for which of the following adverse effects? A. Tremors B. renal calculi C. Dysphagia D. Hearing loss

A. Tremors: concurrent use of sumatriptan and fluoxetine can lead to excessive sitmulation of serotonin receptors, placing the clients at risk for serotonin syndrome. The client can experience tremors, confusion, and hallucinations. Renal calculi: Concurrent use of sumatriptan and fluoxetine does not lead to the formation of renal calculi. Dysphagia: Concurrent use of sumatriptan and fluoxetine does not lead to dysphagia. Hearing loss: Concurrent use of sumatriptan and fluoxetine does not result in hearing loss.

You should instruct a client using phenylephrine spray for nasal congestion to do which of the following to avoid rebound congestion. A. Limit the drugs use to 3 to 5 days B. Add an intranasal glucocorticoid C. Taper the dose before discontinuation D. Restrict the drugs use to one nostril at a time

A. limit drug use to 3 to 5 days Clients should use phenylephrine, a topical sympathomimetic, short-term to prevent rebound congestion. Using an intranasal glucocorticoid, tapering the dose, and instilling the drug in one nostril may help wean the client off a topical sympathomimetic after extended use, but these actions will not prevent rebound congestion. (The Respiratory System Activity)

You are caring for a client who takes acarbose (Precose) and a sulfonylurea to treat type 2 diabetes mellitus. which of the following is an indication of an adverse reactions to this drug combination? A. Polyuria B. Tremors C. Bradycardia D. Thirst

B. Tremors This drug combination can cause hypoglycemia. Indications of a hypoglycemic reaction include hunger, tachycardia, shakiness, tremors, and diaphoresis. Polyuria and thirst are indication of hyperglycemia, not hypoglycemia. Tachycardia, not bradycardia, is an indication of hypoglycemia. Acarbose, with or without sulfonylurea therapy, is unlikely to cause bradycardia. (The Endocrine System Activity)

A provider prescribes IV heparin for this client. Which of the following parameters should you monitor for in regard to determining if a therapeutic dose is being administered? A. INR, 2 to 3 times the clients baseline B. aPTT, 1.5 to 2 times the clients baseline C. Platelet count of 125,000/mm^3 D. PT 11 to 12.5 sec

B. aPTT, 1.5 to 2 times the clients baseline You check the clients activated partial thromboplastin time, or aPTT, every 4 to 6 hours for initial continuous IV heparin therapy. The goal is an aPTT of 1.5 to 2 times the clients baseline for therapeutic anticoagulation. Prothrombin time, or PT, and international normalized ratio, or INR, are parameters that are monitored when clients are on warfarin (Coumadin) therapy, not heparin therapy. A therapeutic INR for warfarin is between 2 and 3 and is not based on the clients baseline. The expected reference rang for PT is from 11 to 12.5 seconds with an increase in this time expected during anticoagulant therapy; however, the INR is the parameter of choice for warfarin. Platelet counts below 100,000/mm^3 warrant discontinuation of heparin therapy. (The Hematologic System Case Study)

Which laboratory test should you monitor for a client who is taking interferon beta-1b to treat multiple sclerosis? (Select all that apply.) A. Serum osmolality B. Blood glucose C. Thyroid hormones D. Liver function E. CBC

C, D, E C. Thyroid hormones D. Liver function E. CBC Interferon beta-1b, an immunosuppressant, may cause hepatotoxicity and myelosuppression. Monitor liver studies and complete blood counts. The client should undergo thyroid function testing every 6 months. This drug is unlikely to alter serum osmolality or glucose levels, but it can elevate serum creatinine and alter serum electrolytes. (The Neurological System Activity (Part 1))

A nurse is collecting data from a client who is asking about taking celecoxib for treatment of joint pain. The nurse should identify that which of the following findings is a contraindication to receiving celecoxib? A. Hyperglycemia B. Allergy to penicillin C. History of myocardial infarction D. Peptic ulcer disease

C. History of myocardial infarction: Celecoxib increases the risk of myocardial infarction caused by increased vasoconstriction and unimpeded platelet aggregation. It is contraindicated for client who has a history of myocardial infarction or heart disease. Hyperglycemia: Celecoxib is not contraindicated for a client who has hyperglycemia. Allergy to penicillin: Celecoxib can cause hypersensitivity reactions in clients who are allergic to sulfonamides or salicylates rather than penicillin. Peptic ulcer disease: Celecoxib should be used with caution for clients who have peptic ulcer disease; however, it is not contraindicated.

A nurse is caring for client who has hyperthyroidism and has been taking methimazole. Which of the following laboratory findings should indicate to the nurse that the medication has a therapeutic effect? A. Decreased blood glucose level B. Increased Hgb C. Increased platelets D. Decreased T4

D. Decreased T4: The nurse should identify that methimazole inhibits the synthesis of thyroid hormone, reducing levels to provide a euthyroid state. Therefore, a decreased level of T4 is an indication of a therapeutic effect. Decreased blood glucose level: The nurse should identify that methimazole does not affect the clients blood glucose level. Therefore, this finding does not indicate a therapeutic effect. Increased Hgb: The nurse should identify that methimazole does not affect the clients Hgb level. Therefore, this finding does not indicate a therapeutic effect. Increased platelets: The nurse should identify that methimazole can cause decreased platelets as and adverse effect. Therefore, this finding does not indicate a therapeutic effect.

You should monitor a client taking dextromethorphan because of the potential for which of the following? A. Tachycardia B. Fluid retention C. Blurred vision D. Drug abuse

D. Drug abuse Dextromethorphan can cause euphoria and hallucination at high doses. Monitor clients taking it for the potential for abuse. Dextromethorphan is unlikely to cause tachycardia, fluid retention, or blurred vision, although it can cause dizziness, dyspepsia, and constipation. (The Respiratory System Activity)

A nurse is reinforcing teaching with a client who is receiving enalapril 20 mg PO daily. The nurse should instruct the client to monitor for which of the following adverse effects of this medication? A. Hypokalemia B. Blurred vision C. Tremors D. Dry cough

D. Dry cough: The nurse should identify that a persistent dry or nonproductive cough is an adverse effect of enalapril. The underlying cause of the dry cough is the accumulation of bradykinin from the medication. The client should notify the provider of this adverse effect. Hypokalemia: The nurse should identify that hyperkalemia is an adverse effect of enalapril. Blurred vision: the nurse should identify that blurred vision is not an adverse effect of enalapril. However, abnormal taste can develop while taking this medication. Tremors: The nurse should identify that tremors are not an adverse effect enalapril. However, dizziness can develop while taking this medication.

The health care professional is about to administer a loading dose a phenytoin IV. Which of the following is essential for the health care professional to do before administering phenytoin IV? A. Protect the phenytoin solution from light. B. Make sure the phenytoin solution is cloudy. C. Mix phenytoin with a dextrose solution. D. Infuse phenytoin slowly.

D. Infuse phenytoin slowly. Health care professionals should administer IV phenytoin slowly, no faster than 50 milligrams per minute, because rapid infusion may cause hypotension, bradycardia, and cardiovascular collapse. Phenytoin solution is not light-sensitive and it is important to discard if it is cloudy or discolored. Phenytoin will precipitate in dextrose solution. (The Neurological System Case Study (Part 1))

A client who is menopausal taking a calcium supplement to prevent osteoporosis. You instruct the client to watch for which of the following indication of hyperglycemia? A. Eye twitching B. Bleeding gums C. Tinnitus D. Nausea

D. Nausea Manifestations of hyperglycemia include anorexia, nausea, vomiting, and constipation. Hypoglycemia can cause twitching, tetany, and muscle spasms. Vitamin C deficiency can cause bleeding gums and gingivitis. (The Musculoskeletal System Activity)

For which of the following clients should a health care provider question the use of sildenafil to treat erectile dysfunction. A client who is taking: A. Oxybutynin chloride for urinary incontinence B. Phenytoin for seizure disorder C. Thyroid for hypothyroid D. Nitroglycerin as needed for chest pain

D. Nitroglycerin as needed for chest pain Severe hypotension may result if nitroglycerin and sildenafil are used together. Avoid concurrent use. Monitor for hypotension. Sildenafil may be used in clients taking oxybutynin chloride, phenytoin, or thyroid. (The Reproductive and Genitourinary Systems Activity)

A nurse is reviewing the history of a client who is to start taking cefotetan to treat a bacterial infection. Which of the following information from the client's medical record should the nurse report to the provider before the client begins receiving this medication? A. Hearing impairment B. Milk-protein allergy C. Tendon pain D. Penicillin allergy

D. Penicillin allergy: Cefotetan is a cephalosporin, an antibiotic structurally similar to penicillins. The client who ahs a severe allergy to penicillin can develop cross-reactivity and have an allergic reaction to cephalosporins. Hearing impairment: Hearing loss is a contraindication for some types of antibiotics, such as aminoglycosides, due to their ototoxic effects. Milk-protein allergy: Cefditoren, a cephalosporin, can cause an allergic reaction in clients who have milk-protein hypersensitivity. Tendon pain: Clients who have tendon pain should use fluoroquinolone antibiotics with caution due to the possible adverse effect of tendon rupture.

Which of the following drugs should you have ready in case of heparin overdose? A. Aminocaproic acid B. Deferoxamine C. Vitamin K D. Protamine

D. Protamine Protamine reverses the effects of heparin. For an overdose, the healthcare professional should stop heparin and administer protamine no faster than 20 mg/min. Aminocaproic acid, a coagulator, acts by inhibiting fibrinolysin. It can stop excessive bleeding due to alteplase. A chelating agent, such as deferoxamine, treats iron toxicity. Vitamin K reverses the effects of warfarin. (The Hematologic System Case Study)

A nurse is caring for a client who is taking disulfiram and confused alcohol 12 hr ago. Which of the following adverse reactions is the priority finding to report to the provider? A. Hyperemesis B. Severe headache C. Palpitations D. Respiratory depression

D. Respiratory depression: When using urgent vs. nonurgent approach to client care, the nurse should determine that the priority finding is respiratory depression, which can indicate the client is experiencing acetaldehyde syndrome, a life-threatening event. Hyperemesis: Hyperemesis is nonurgent because it is an expected reaction following alcohol consumption for a client who is taking disulfiram. Therefore, another finding is the priority. Severe headache: Severe headache is nonurgent because it is an expected reaction following alcohol consumption for a client who is taking disulfiram. Therefore, another finding is the priority. Palpitations: Palpitations are nonurgent because there an expected reaction following alcohol consumption for a client who is taking disulfiram. However, another finding is the priority.

A nurse is reinforcing teaching with a client who has type 1 diabetes mellitus and is learning to self-administer NPH insulin. Which of the following clients actions indicates an understanding of the teaching? A. The client shakes the insulin vial prior to drawing up the dose. B. The client uses the tip of the plunger to measure the correct dose. C. The client injects air into the vial after inverting it. D. The client wipes the cap with alcohol prior to filling the syringe.

D. The client wipes the cap with alcohol prior to filling the syringe.: The client should wipe the cap with alcohol prior to filling the syringe to prevent contamination. The client shakes the insulin vial prior to drawing up the dose.: The client should gently roll the vial of NPH insulin between his palms to disperse the particles in the solution. The client uses the tip of the plunger to measure the correct dose.: In order to draw up the correct dose of medication, the client should use the widest portion of the plunger and line it up with with prescribed number on the syringe. The client injects air into the vial after inverting it.: Injecting air into the vial prior to inverting it will prevent contaminating the needle.

A nurse is collecting data from a client who has been taking digoxin for 1 month. The nurse should identify which of the following findings as a manifestation of digoxin toxicity? A. Pulse rate of 100/min B. Blood pressure 140/90 mmHg C. Wheezing D. Vomiting

D. Vomiting: The nurse should identify vomiting as an early manifestation of digoxin toxicity. Pulse rate of 100/min: A pulse rate of 100/min is within the expected reference range and is not a manifestation of digoxin toxicity. Blood pressure 149/90 mmHg: A blood pressure 140/90 mmHg is above the expected reference range. However, hypotension, rather than hypertension, is a manifestation of digoxin toxicity. Wheezing: Wheezing is not a manifestation of digoxin toxicity.

A nurse is planning to administer metoprolol to a client who has heart failure and a heart rate of 48/min. Which of the following actions should the nurse take? A. Ambulate the client before administering the medication. B. Give the medication when the client has an empty stomach. C. Administer one-half of the clients prescribed dose. D. Withhold the clients medication.

D. Withhold the clients medication.: The nurse should withhold the metoprolol when the clients heart rate is 50/min or less and notify the provider. Ambulate the client before administering the medication.: A client who has a heart rate of 50/min or below can become hypotensive. Therefore, ambulating can increase the clients risk of falling. However, the nurse should not administer this medication because of the clients bradycardia. Give the medication when the client has an empty stomach.: The nurse should administer the mediation to the client with meals or immediately after meals. This medication can mask manifestations of hypoglycemia if the client has diabetes. Administer one-half of the clients prescribed dose.: The nurse should notify the provider of the clients heart rate to determine when to administer the next dose or if a decrease in dosage is needed.

A nurse is caring for a client who is receiving 0.9% sodium chloride 1,000 mL to infuse over 8 hr. The drop factor on the manual IV tubing is 15 gtt/mL. The nurse should ensure that the manual infusion is set to deliver how many gtt/min? (Round the answer to the nearest whole number. Use a leading zero if it applies. Do not use a trailing zero.) Answer is fill in the blank.

Measurement the nurse should calculate? gtt/min Quantity of the drop factor that is available? 15 gtt/mL Total infusion time? 8 hr 8hrx60 min = 480 1000 mL x 15 gtt/mL / 480 min = 31.25 x= 31 gtt/min

At 0800 hr a nurse initiates a 1,000 mL intravenous (IV) infusion on a client, which is running at 125 mL/hr. It is now 1300 hr. How much fluid is left in the IV bag? (Round the answer to the nearest whole number. Use leading zero if it apples. Do not use trailing zero.) Answer is fill in the blank

Unit = mL Total volume = 1,000 mL Rate = 125 mL/hr Time = 5hr Covert units? No. Rate (mL/hr) x Time (hr) = Volume infused (X mL) 125 mL/hr x 5 hr = X mL Volume infused 625 mL Total volume (mL) - Volume infused (mL) = Volume remaining (mL) 1000 mL - 625 mL = 375 mL Round if necessary? No Reassess to determine whether the amount remaining makes sense? It does. Answer = 375 mL

A nurse is preparing to administer cefazolin 1 g in 100 mL 0.9% sodium chloride to infuse over 30 min. The drop factor of the manual IV tubing is 15 gtt/mL. The nurse should set up the manual IV infusion to deliver how many gtt/min? (Round the answer to the nearest whole number use a leading zero if it applies. Do not use a trailing zero.) Answer is fill in the blank.

Unit of measurement the nurse should calculate: gtt/min Volume to infuse: 100 min Infusion time: 30 min 100 mL x 15 gtt / 30 min = 50 X=50

A nurse is reviewing a clients medical history before administering hydromorphone for postoperative pain. The nurse should notify the provider of which of the following findings before administering this medication? A. Benign prostatic hyperplasia B. History of hypertension C. Rheumatoid arthritis D. Allergy to aspirin

A. Benign prostatic hyperplasia: A client who has benign prostatic hyperplasia is at increased risk for developing acute urinary retention while taking opioids. Therefore, the nurse should notify the provider about this finding before administering hydromorphone. History of hypertension: a client who has history of hypertension can experience a lowering of blood pressure when taking an opioid due to vasodilation effects. Rheumatoid arthritis: Rheumatoid arthritis is not a contraindication for receiving hydromorphone. History of severe pulmonary disease can be a contraindication for hydromorphone. Allergy to aspirin: An allergy to aspirin is not a contraindication for receiving hydromorphone. History of an allergy to bisulfates, a component in some medications, is a contraindication for hydromorphone.

The health care professional instructs a client about the therapeutic actions of sucralfate. She explains that this drug promotes ulcer healing by which action? A. Creates a protective barrier B. Increases gastric pH C. Inhibits the proton pump D. Neutralizes gastric acid

A. Creates a protective barrier Sucralfate promotes ulcer healing by forming a protective barrier that coats the ulcer. This coating protects the ulcer form further erosion from acid and enzymes and allows the ulcer to heal. Antacids, such as aluminum hydroxide, increase gastric pH and neutralize stomach acid. Omeprazole reduces gastric acid secretion by inhibiting the aid pump. (The Gastrointestinal System Case Study)

A nurse is collecting data from a client who has Parkinson's disease and is taking levodopa/carbidopa. The nurse should identify which of the following findings as an adverse effect of this medication? A. Dark urine B. Hypertension C. Increased salivation D. Bradycardia

A. Dark urine: The nurse should identify that levodopa/carbidopa can cause a darkening of the clients urine, sweat, and saliva. Hypertension: The nurse should identify that levodopa/carbidopa can cause orthostatic hypotension. Increased salivation: The nurse should identify that levodopa/carbidopa can cause dry mouth. Bradycardia: The nurse should identify that levodopa/carbidopa can cause tachycardia.

A nurse is assisting with the admission of a client who reports muscle aches and pains associated with construction work. The provider suggests taking acetaminophen. The nurse should notify the provider about which of the following information obtained from the client? A. History of hepatitis B B. Cigarette smoker C. History of hypertension D. Allergy to sulfonamides

A. History of hepatitis B: Acetaminophen can cause hepatoxicity. Clients who have a history of liver damage are at risk for further liver damage and can require a lower dose. Cigarette smoker: Clients who smoke cigarettes can take acetaminophen. History of hypertension: Clients who have hypertension can take acetaminophen because it does not have an adverse effect on the cardiovascular system. Allergy to sulfonamides: There is no cross-sensitivity between acetaminophen and sulfa. Therefore, clients who have a sulfa allergy can take acetaminophen.

A nurse is reinforcing dietary teaching with a client who has new prescription for phenelzine. Which of the following foods should the nurse include in the teaching as a appropriate food choice? A. Yogurt B. Avocado C. Smoked salmon D. Peperoni

A. Yogurt: Clients taking phenelzine should avoid consuming tyramine, which can cause high blood pressure. Yogurt contains little or no tyramine. Therefore, it is an appropriate food choice to include in the teaching. Avocado: Clients taking phenelzine should avoid consuming avocados, which have a high tyramine content. Smoke salmon: Clients taking phenelzine should avoid consuming smoked salmon, which has high tyramine content. Pepperoni: Clients taking phenelzine should avoid consuming peperoni, which has high tyramine content.

The client is taking oral contraceptives for birth control. Which of the following instructions should the health care professional include? A. It is safe to discontinue birth control while taking phenytoin. B. Additional birth control methods may be needed. C. Oral contraceptives may increase the risk for phenytoin toxicity. D. Oral contraceptives may reduce the effectiveness of phenytoin.

B. Additional birth control methods may be needed. Phenytoin is teratogenic and cause cause serious birth defects, so women taking it should prevent pregnancy. Phenytoin can reduce the effectiveness of oral contraceptives, so the dosage may have to be increased or other birth control methods used or added. Diazepam, omeprazole, and valproic acid can increase serum phenytoin levels. Carbamazepine and phenobarbital can decrease phenytoin levels. (The Neurological System Case Study (Part 1))

A nurse is reinforcing teaching with a client who has a new prescription for ethinyl estradiol/norethindrone, an oral contraceptive. Which of the following client statements indication to the nurse an understanding of the teaching? A. "I should expect my menstrual flow to increase?" B. "I should monitor my blood pressure for hypotension while on this medication." C. "I will take the medication at the same time each day." D. "This type of medication is the most effective because it only contains estrogen."

C. "I will take the medication at the same time each day.": Th client should take this medication at the same time each day to maintain a consistent level to reduce fertility and the chance of pregnancy. "I should expect my menstrual flow to increase?": The nurses should reinforce with the client that the volume of her menstrual flow will decrease, as well as the number of days of menses. "I should monitor my blood pressure for hypotension while on this medication.": the client should monitor her blood pressure for hypertension because the medication causes increased secretion of aldosterone and angiotensin. "This type of medication is the most effective because it only contains estrogen.": The nurse should reinforce the client that ethinyl estradiol/norethindrone is a combination oral contraceptive with each tablet containing both estrogen and progestin.

Which of the following drugs is appropriate for this client at this time? (Video - patient arrives at the ER, feeling short of breathe for the last 30 minutes, O2 stat 88%, B/P 142/88, P 124) A. Formoterol B. Theophylline C. Albuterol D. Salmeterol

C. Albuterol Albuterol is a short-acting beta2 agonist that you administer to relieve acute bronchospasm. Formoterol and salmeterol are long-acting beta2 agonists you give every 12 hours to prevent bronchospasm. Theophylline is a long-acting methylxanthine to prevent bronchospasm. (The Respiratory System Case Study)

A client about to start taking somatropin (Genotropin). You plan to evaluate the effectiveness of this drug therapy with which of the following assessments. A. Level of consciousness B. ECG C. Height and weight D. Breath sounds and respiratory rate

C. Height and weight Somatropin, a growth hormone, increases growth in clients who have insufficient growth hormone. Gradual increases in weight and height reflect effective therapy, so you'll monitor the clients weight and height on an ongoing basis. Somatropin is unlikely to affect heart rate or rhythm, level of consciousness, or respiratory status. It can however, cause hyperglycemia and hypercalciuria. be sure to monitor serum glucose and urine calcium and instruct the client to watch for and report polyphagia, polyuria, or flank pain. (The Endocrine System Activity)

A nurse is caring for a client who has chronic kidney disease and has been receiving epoetin alfa for 2 weeks. Which of the following findings should indicate to the nurse that the clients medication is having the desired therapeutic effect? A. Albumin is within the expected reference range. B. Urine output increases to 60 mL/hr. C. Hemoglobin rises 0.5 g/dL. D. Blood urea nitrogen level is within the expected reference range.

C. Hemoglobin rises 0.5 g/dL: Initial therapeutic effects, such as hemoglobin rising 0.5 g/dL, can occur within the first 2 weeks of therapy. The clients hemoglobin should reach target levels (10 to 11 g/dL) in 2 to 3 months. Albumin is within the expected reference range: Epoetin alfa does not affect the clients albumin level. Urine output increases to 60 mL/hr: Epoetin alfa does not affect urine output. Blood urea nitrogen level is within the expected reference range: Epoetin alfa does not affect the clients BUN level.

A primary care provider prescribes calcitonin salmon for a post-menopausal client. Knowing the adverse effects of calcitonin, you instruct the client to A. Increase fluid intake. B. Rise slowly from a reclining position. C. Increase calcium and vitamin D intake. D. Rest painful joints after exercise.

C. Increase calcium and vitamin Calcitonin can cause hypoglycemia due to increased excretion of calcium. The client should increase calcium and vitamin D intake and watch for indications of hypoglycemia, such as muscle cramps and numbness in the fingers or toes. Increasing fluid intake will not prevent adverse effects: urinary frequency. Calcitonin does not cause orthostatic hypotension, although it can cause weakness and shortness of breath. Calcitonin is more likely to cause headache than joint pain. (The Musculoskeletal System Activity)

A nurse is collecting data from a client who is taking tobramycin. Which of the following findings should the nurse report to the provider immediately? A. Report of nausea B. Fever C. Oliguria D. Report of headache

C. Oliguria: Oliguria indicates the clients is at greatest risk for nephrotoxicity. Therefore, the nurse should report this finding to the provider immediately. Report of nausea: The nurse should report nausea as an adverse effect of tobramycin and implement interventions to prevent vomiting. However, another finding is the priority to report to the provider. Fever: The nurse should report a fever as a possible indication of superinfection and determine whether further diagnostic testing is required. However, another finding is the priority to report to the provider. Report of headache: The nurse should report a headache as a possible manifestation of an electrolyte imbalance and obtain a prescription for analgesia. However, another finding is the priority to report to the provider.

A nurse is reviewing the medical record of a client who has a new prescription for cephalexin to treat pneumonia. which of the following data should the nurse report to the provider before the client receives this medication? A. Neomycin sensitivity B. Egg allergy C. Penicillin allergy D. Alcohol sensitivity

C. Penicillin allergy: The nurse should identify that cephalosporin, a classification of antibiotics whose structure is similar to that of penicillin. Although it is rare, a life-threatening cross-reactivity can develop in clients who are allergic to penicillin and take cephalosporin. the nurse should notify the provider. Neomycin sensitivity: The nurse should identify that neomycin sensitivity is a contraindication to receiving the measles, mumps, and rubella (MMR) immunizations. However, it is not a contraindication to receiving cephalexin. Egg allergy: The nurse should identify that a previous anaphylactic reaction to eggs requires a risk evaluation before receiving the influenza immunization. However, it is not a contraindication to receiving cephalexin. Alcohol sensitivity: The nurse should identify that alcohol intolerance or sensitivity is a contraindication to receiving sirolimus. However, it is not a contraindication to receiving cephalexin.

You are reviewing the history of a client who is about to start therapy with factor VIII to treat hemophilia A. Which of the following drugs the client takes should alert you to take further action? A. Albuterol B. Phenytoin C. Phenelzine D. Aspirin

D. Aspirin Clients who have hemophilia should not take aspirin or NSAIDs because they increase the risk for bleeding. They may take albuterol, phenytoin, and phenelzine. A second-generation NSAID, such as celecoxib, would pose less risk for bleeding. (The Hematologic System Activity)

A nurse is monitoring a client who is receiving repaglinide for type 2 diabetes mellitus. Which of the following laboratory tests should the nurse plan to review to obtain information about the long-term therapeutic effect of this medication? A. Fasting blood glucose level B. 1-hr oral glucose tolerance test C. Urinary ketones D. Glycosylated HbA1c

D. Glycosylated HbA1c: The clients HbA1c value measures the average of blood glucose levels over the past 2 to 3 months. Therefore, the nurse should review this laboratory test to obtain information about the long term therapeutic effect of repaglinide. Fasting blood glucose level: The fasting blood glucose level indicates the clients current status, not the long-term therapeutic effect of repaglinide. 1-hr oral glucose tolerance test: The 1-hr glucose tolerance test evaluates the clients response to a carbohydrate challenge. The client can undergo this test to determine if they have gestational diabetes. Urinary ketones: The presence of urinary ketones indicates diabetic ketoacidosis, not the long-term therapeutic effect of repaglinide.

When talking with a client about self-administering regular insulin (Humulin R), you should include which of the following instructions? A. Shake the vial vigorously B. Expect the solution to appear cloudy C. Store unopened vials at room temperature D. Inject the insulin subcutaneously

D. Inject the insulin subcutaneously Make sure the client understands how to inject insulin subcutaneously. Tell him or her not to shake the vial vigorously but to rotate it gently to disperse the particles. Do not use insulin that appears cloudy or discolored. Remind them to refrigerate unopened vials until their expiration date and opened vials can be left out at room temperature up to 1 month. (The Endocrine System Activity)

You are administering lidocaine, 50 mg via IV bolus, to the client with ventricular dysrhythmia. You monitor the client for which adverse reactions to lidocaine? A. Metabolic alkalosis B. Hypertension C. Tachycardia D. Paresthesias

D. Paresthesias High doses of lidocaine, a class 1B antidysrhythmic, may cause numbness and tingling, which may lead to seizure activity. Lidocaine is not likely to cause metabolic alkalosis. High doses of the drug cause hypotension and bradycardia, not hypertension and tachycardia. (The Cardiovascular System Activity)

Before administering IV folic acid, you should explain to the client that this drug can cause which of the following adverse effects? A. Headache B. Nausea C. Dizziness D. Rash

D. Rash IV folic acid can cause a rash and irritation with injection, so tell the client to expect these effects. IV folic acid is unlikely to cause nausea. Ferrous sulfate is an example of a hematologic drug that can cause nausea and constipation. IV folic acid is unlikely to cause nausea and constipation. IV folic acid is unlikely to cause headache or dizziness. Alteplase, a thrombolytic, is an example of a hematologic drug that can cause a headache. Oprelvekin, a thrombolytic growth factor, is an example of a hematologic drug that can cause dizziness, headache, and nervousness. (The Hematologic System Activity)

Match the therapeutic use 1. Dinoprostone 2. Terbutaline 3. Human chorionic gonadotropin 4. Medroxyprogesterone A. Induces uterine contractions B. Treats endometriosis C. Stops uterine contractions D. Stimulates ovulation

1. > A, 2. > C, 3. > D, 4. > B Dinoprostone: Induces uterine contractions Terbutaline: Stops uterine contractions Human chorionic gonadotropin: Stimulates ovulation Medroxyprogesterone: Treats endometriosis Dinoprostone, a synthetic prostaglandin, promotes cervical ripening before labor induction and stimulates uterine contractions are cervical ripening. Terbutaline, a beta2-adrenergic agonist, inhibits uterine contractions and stops preterm labor. Human chorionic gonadotropin, a polypeptide hormone, promotes ovulation after induction of follicular maturation. Medroxyprogesterone, a progestin, treats endometriosis and palliates endometrial cancer. (The Reproductive and Genitourinary Systems Activity)

Match with therapeutic use 1. Radioactive iodine-131 (Iodotope) 2. Glucagon (GlucaGen) 3. Desmopressin (DDAVP) 4. Glipizide (Glucotrol) A. Hypoglycemia B. Thyroid cancer C. Diabetes insipidus D. Type 2 diabetes mellitus

1. > B, 2. > A, 3. > C, 4. > D Radioactive iodine-131 (Iodotope): Thyroid cancer Glucagon (GlucaGen): Hypoglycemia Desmopressin (DDAVP): Diabetes insipidus Glipizide (Glucotrol): Type 2 diabetes mellitus Desmopressin, an antidiuretic hormone, treats diabetes insipidus. Glucagon, a hyperglycemic drug, treats hypoglycemia due to an insulin overdose. Glipizide, a sulfonylurea, is an oral antidiabetic drug that treats type 2 diabetes mellitus. Radioactive iodine-131, an antithyroid drug, treats hyperthyroidism from Graves' disease and also thyroid cancer. (The Endocrine System Activity)

Match the pharmacokinetic process. 1. Metabolism 2. Absorption 3. Excretion 4. Distribution A. The removal of the drug or its metabolites through the kidneys, gastrointestinal tract, skin, or lungs B. The change of a drug into a more or less potent or more soluble form after it passes through the liver, kidneys, intestinal mucosa, or other body part C. The movement of a drug by the circulatory system to its intended site of action D. The movement of a drug from the site of administration into the circulatory system

1. > B, 2. > D, 3. > A, 4. > C. Metabolism: The change of a drug into a more or less potent or more soluble form after it passes through the liver, kidneys, intestinal mucosa, or other body part Absorption: The movement of a drug by the circulatory system to its intended site of administration into the circulatory system Excretion: The removal of the drug or its metabolites through the kidneys, gastrointestinal tract, skin, or lungs Distribution: The movement of a drug by the circulatory system to its intended site of action Distribution is the movement of a drug by the circulatory system to its intended site of action. Metabolism is the change that occurs in a drug into more or less potent or more soluble form after it passes through the liver, kidneys, intestinal mucosa, or other body organ or part. Excretion is the removal of the drug or its metabolites through the kidneys, gastrointestinal tract, skin, or lungs. Absorption is the movement of a drug from the site of the administrations into the circulatory system. (Introduction into Pharmacology Activity)

Match with pharmacologic action 1. Filgrastim 2. Epoetin alfa 3. Alteplase 4. Clopidogrel A. Stimulates RBC production B. Dissolves blood clots C. Stimulates leukocyte production D. Inhibits platelet aggregation

1. > C, 2. > A, 3. > B, 4. > D Filgrastim: Stimulates leukocyte production Epoetin alfa: Stimulates RBC production Alteplase: Dissolves blood clots Clopidogrel: Inhibits platelet aggregation Filgrastim, a leukopoietic growth factor, stimulates leukocyte production for clients who are myelosuppressed due to chemotherapy or disease. Alteplase, a thrombolytic drug, dissolves a thrombus or blood clot that is already formed. Epoetin alfa stimulates RBC production in the bone marrow. It treats anemia from renal failure, malignancies, or AIDS. Clopidogrel, an antiplatelet drug, inhibits platelet aggregation and reduces thrombus formation. (The Hematologic System Activity)

Match the drug classification with the therapeutic use 1. Hydantoin 2. Dopamine receptor agonist 3. Immunomodulator 4 Cholinesterase inhibitor A. Reduces the effects of Parkinson's disease B. Reduces the symptoms of multiple sclerosis C. Controls seizures D. Helps minimize symptoms of Alzheimer's disease

1. > C, 2. > A, 3. > B, 4. > D Hydantoin: Controls seizures Dopamine receptor agonist: Reduces the effects of Parkinson's disease Immunomodulator: Reduces symptoms of multiple sclerosis Cholinesterase inhibitor: Helps minimize symptoms of Alzheimer's disease The hydantoins control tonic-clonic seizures, as well as partial seizures. They act by decrease the neuronal activity of seizure-generating cells in the brain by inhibiting the influx of sodium through sodium channels. Immunomodulators help reduce the symptoms of multiple sclerosis. They seem to inhibit the movement of leukocytes, which are produced as a part of the body's defective autoimmune response, and thus protect the myelin sheath of neurons from their damaging effects. Cholinesterase inhibitors help improve cognitive function in clients with mild to moderate Alzheimer's disease, thus minimizing symptoms. They increase the amount of acetylcholine depletion that characterizes Alzheimer's disease. Dopamine receptor agonists, also known as monoamine oxidase, or MAO, Inhibitors, are one of the types of drugs used to control the symptoms of Parkinson disease. Dopamine-replacement drugs are another drug classification that comprises pharmacologic therapy for Parkinson disease. These drugs either mimic the action of dopamine or make more of it available and thus counteract the dopamine depletion that characterizes Parkinson disease. (The Neurological System Activity (Part 1))

Match with therapeutic effect 1. Omeprazole (Prilosec) 2. Sulfasalazine (Azulfidine) 3. Dimenhydrinate 4. Metoclopramide (Reglan) A. Inflammatory bowel disease B. Motion Sickness C. Duodenal ulcer D. Nausea, vomiting

1. > C, 2. > A, 3. > B, 4. > D Omeprazole (Prilosec): Duodenal ulcer Sulfasalazine (Azulfidine): Inflammatory bowel disease Dimenhydrinate: Motion sickness Metoclopramide (Reglan): Nausea, vomiting Dimenhydrinate helps prevent and teat motion sickness. It also treats vertigo and reduces nausea and vomiting from radiation sickness. Sulfasalazine, an anti-inflammatory drug, treats inflammatory bowel disease and ulcerative colitis. Metoclopramide, a prokinetic, reduces gastrointestinal motility and nausea. Omeprazole, a proton pump inhibitor, reduces gastric acid secretion and treats duodenal and gastric ulcers, prolonged dyspepsia, gastrointestinal reflux disease, and erosive esophagitis. (The Gastrointestinal System Activity)

Match with therapeutic use 1. Ranitidine 2. Azathioprine 3. Docusate sodium 4. Bisacodyl A. Inflammatory bowel disease B. Stool softener C. Constipation D. Gastrointestinal reflux disease (GERD)

1. > D, 2. > A, 3. > B, 4. > C Ranitidine: Gastrointestinal reflux disease (GERD) Azathioprine: Inflammatory bowel disease Docusate sodium: Stool softener Bisacodyl: Constipation Bisacodyl, a stimulant laxative, reduces constipation by increasing peristalsis and increasing the amount of intestinal fluid. Docusate sodium is a stool softener. Ranitidine, a histamine H2 antagonist, inhibits gastric acid secretion. It treats gastric and duodenal ulcers, relieves heartburn and dyspepsia, and helps minimize the symptoms of GERD. Azathioprine, an immunosuppressants, reduces the inflammation of inflammatory bowel disease (unlabeled use). (The Gastrointestinal System Activity)

Match the drugs with the corresponding classification and action 1. Verapamil (Calan), 2. Quinidine, 3. Propranolol (Inderal), 4. Amiodarone (Cordarone) A. Sodium channel blocker that treats supraventricular and ventricular dysrhythmias B. Beta adrenergic blocker that controls atrial and ventricular tachycardia C. Potassium channel blocker that treats ventricular fibrillation and ventricular tachycardia D. Calcium channel blocker that treats supraventricular tachycardia and atrial fibrillation or flutter

1. > D, 2. > A, 3. > B, 4. > C Verapamil (Calan): Calcium channel blocker that treats supraventricular tachycardia and atrial fibrillation or flutter Quinidine: Sodium channel blocker that treats supraventricular and ventricular dysrhythmias Propranolol (Inderal): Beta adrenergic blocker that controls atrial and ventricular tachycardia Amiodarone (Cordarone): Potassium channel blocker that treats ventricular fibrillation and ventricular tachycardia Quinidine is a sodium channel blocker that treats supraventricular and ventricular dysrthmias. Propranolol is a beta-adrenergic blocker that controls atrial and ventricular tachycardia. Amiodarone is a potassium channel blocker that treats ventricular fibrillation and ventricular tachycardia. Verapamil is a calcium channel blocker that treats supraventricular tachycardia and atrial fibrillation or flutter. (The cardiovascular System Activity)

Match the pharmacologic term. 1. Peak drug level 2. Trough drug level 3. onset of action 4. Duration of action A. Point in the time when a drug is at its lowest level in the body B. The time it takes for a drug to demonstrate a therapeutic response C. The length of time the drugs therapeutic effect lasts. D. The length of time the drugs therapeutic effects lasts.

1. > D, 2. > A, 3. > B, 4. > C. Peak drug level: The time it takes a drug to demonstrate its full therapeutic effect Through drug level: The point in time when a drug is at its lowest level in the body Onset of action: The time it takes for a drug to demonstrate a therapeutic response Duration of action: The length of time the drugs therapeutic effect lasts The peak drug level is the time it takes a drug to demonstrate its full therapeutic effect. Duration of action is the length of time the drugs therapeutic effect lasts. The trough drug level is the point in time when a drug is at its lowest level in the body. The onset action is the time it takes for a drug to demonstrate a therapeutic response. (Introduction into Pharmacology Activity)

Match the drug used to treat osteoporosis 1. Bisphosphonates 2. Calcium supplements 3. Calcitonin 4. Selective estrogen receptor modulators A. Increases the excretion of calcium B. Raises calcium levels C. Mimics the effects of estrogen on the bones D. Reduces the number of osteoclasts

1. > D, 2. > B, 3. > A, 4. > C Bisphosphonates: Reduces the number of osteoclasts Calcium supplements: Raises calcium levels Calcitonin: Increases the excretion of calcium Selective estrogen receptor modulators: Mimics the effects of estrogen on the bones Selective estrogen receptors, called SERMs, treat postmenopausal osteoporosis by mimicking the effects of estrogen on the bones. Bisphosphonates treat age-related and glucocorticoid-related osteoporosis by reducing the number of osteoclasts. Calcitonin acts similarly to the calcitonin the thyroid glands produce by increasing the excretion of calcium. calcium supplements raise calcium levels and also help neutralize gastric acid. (The Musculoskeletal System Activity)

Match with therapeutic use 1. Hydrocortisone 2. Propylthiouracil 3. Somatropin 4. Levothyroxine A. Turner's syndrome B. Thyrotoxic crisis C. Hypothyroidism D. Addison's disease

1. > D, 2. > B, 3. > A, 4. > C Hydrocortisone: Addison's disease Propylthiouracil: Thyrotoxic crisis Somatropin: Turner's syndrome Levothyroxine: Hypothyroidism Hydrocortisone, a glucocorticoid, treats acute and chronic adrenocortical insufficiency. Somatropin, a growth hormone, treats growth hormone, treats growth hormone deficiencies, such as Turner's syndrome. Propylthiouracil, an antithyroid drug, treats hyperthyroidism or Graves' disease and thyrotoxic crisis. Levothyroxine, a thyroid replacement hormone, treats hypothyroidism. (The Endocrine System Activity)

Match the therapeutic use 1. Testosterone 2. Finasteride 3. Oxybutynin chloride 4. Sildenafil A. Urinary incontinence B. Erectile dysfunction C. Benign prostatic hypertrophy D. Male puberty induction

1. > D, 2. > C, 3. > A, 4. > B Testosterone: Male puberty induction Finasteride: Benign prostatic hypertrophy Oxybutynin chloride: Urinary incontinence Sildenafil: Erectile dysfunction Finasteride, a 5-alpha reductase inhibitor, treats benign prostatic hypertrophy and male pattern baldness. Oxybutynin, an anticholinergic, reduces urinary incontinence and helps manage symptoms of neurogenic bladder. Sildenafil, a PDE-5 inhibitor, improves sexual response for men who have erectile dysfunction. Testosterone, an androgen, treats hypogonadism and delayed puberty in males and provides palliative treatment of breast cancer in women. (The Reproductive and Genitourinary Systems Activity)

Prior to starting alendronate, the nurse should assess which of the following in the client? (Select all that apply.) A. Ability to swallow B. Baseline vision C. The clients ability to lay flat d. Absence of nasal dryness

A, B A. Ability to swallow B. Baseline vision Alendronate is a contraindication in clients who have difficulty swallowing due to the risk of esophagitis. Alendronate can cause vision changes in some clients, so it would be important to know the clients baseline vision to assess for changes during drug therapy. The client must be able to tolerate sitting in an upright position for 30 minutes following administration of alendronate. Alendronate is not administered intranasally and does not affect the clients nostrils. (The Musculoskeletal System Activity)

A client is about to start taking sulfasalazine to treat inflammatory bowel disease. You should instruct the client to watch for and report which of the following adverse drug reactions of the drug? (Select all that apply.) A. Sore throat B. Fever C. Joint pain D. Constipation E. Dry mouth

A, B A. Sore throat B. Fever Sulfasalazine, an anti-inflammatory drug, can cause blood dyscrasias, such as agranulocytosis and hemolytic and macrocytic anemia. Instruct clients to avoid crowds and people who have communicable infections and to report sore throat, fever, or other indications of infection. Sulfasalazine doesnt typically cause arthralgia. Sulfasalazine is more likely to cause diarrhea than constipation. It is unlikely to cause dry mouth. Loperamide is an example of an antidiarrheal drug that causes dry mouth. (The Gastrointestinal System Activity)

You are instruct a school-age child and his parent about the use of amphetamine-dextroamphetamine to manage the childs attention-deficit/hyperactivity disorder. Which clinical manifestations should the nurse instruct the parent to report? (Select all that apply.) A. Weight loss B. Insomnia C. Hallucinations D. Orthostatic hypotension E. Flu-like symptoms

A, B, C A. Weight loss B. Insomnia C. Hallucinations Amphetamine-dextroamphetamine, a central nervous system stimulant, can lead to weight loss. Clients should take the drug right before meals and maintain healthful eating habits. Amphetamine-dextroamphetamine can cause insomnia and restlessness, so clients should take the drug in the morning. It can cause paranoid psychosis; thus, clients should report delusions, hallucinations, and changes in mood. Risperidone, an atypical antipsychotic drug, is likely to cause orthostatic hypotension. Atomoxetine may cause liver injury. Clients taking atomoxetine should report signs of live injury such as flu-like symptoms, yellowing skin, and abdominal pain. (The Neurological System Activity (Part 1))

A client who has a peanut allergy comes to the emergency department with suspected anaphylactic shock. Which of the following actions should you anticipate taking? (Select all that apply.) A. Initiate oxygen therapy to support respiratory function. B. Administer epinephrine to increase blood pressure. C. Give diphenhydramine to stop histamine release. D. Establish and maintain an open airway to ensure oxygenation. E. Administer a drug agonist to neutralize the allergic reaction

A, B, C, D A. Initiate oxygen therapy to support respiratory function. B. Administer epinephrine to increase blood pressure. C. Give diphenhydramine to stop histamine release. D. Establish and maintain an open airway to ensure oxygenation. When a client has an anaphylactic reaction, your priority is to establish an open airway and then administer oxygen immediately. Give epinephrine to increase blood pressure and diphenhydramine to stop the release of histamine. Do not administer a drug agonist, because it would elicit an action in the body instead of blocking an action. (Introduction to Pharmacology Activity)

A female client of childbearing potential is starting therapy with misoprostol to prevent a gastric ulcer. Which of the following instructions should you include when teaching this client? A. Mid-cycle spotting can occur B. Take a pregnancy test prior to therapy C. Report excessive menstrual pain D. Use effective contraception E. Avoid taking the drug at bedtime

A, B, C, D A. Mid-cycle spotting can occur B. Take a pregnancy test prior to therapy C. Report excessive menstrual pain D. Use effective contraception Misoprostol, a prostaglandin E analog, is a drug with teratogenic effects. It can induce uterine contractions and abort a pregnancy. Instruct females of childbearing age to confirm that they are not pregnant before taking this drug and to use effective contraception throughout drug therapy. Provide oral and written information about the risks of taking misoprostol during pregnancy. This drug can cause dysmenorrhea, so be sure to monitor for excessive menstrual pain or bleeding. Clients take this drug four times a day, with meals and again at bedtime. (The Gastrointestinal System Activity)

A nurse is reinforcing teaching with a female client who has a new prescription for isotretinoin. Which of the following instructions should the nurse include in the teaching? (Select all the apply.) A. "You will need to have your liver enzymes monitored after 1 month." B. "You can have nosebleeds while taking the medication." C. "You should report any thoughts for harming yourself." D. "You will need to have two negative pregnancy tests prior to starting the medication." E. "You will need to take a vitamin A supplement twice daily."

A, B, C, D A. "You will need to have your liver enzymes monitored after 1 month.": The client should have her liver enzymes monitored 1 month after therapy and periodically thereafter because isotretinoin is metabolized in the liver. B. "You can have nosebleeds while taking the medication.": Due to the drying effects of isotretinoin, nosebleeds are very common. C. "You should report any thoughts for harming yourself.": Isotretinoin can cause depression, which can lead to suicide. The client or the clients family should report these thoughts to the provider. D. "You will need to have two negative pregnancy tests prior to starting the medication.": Due to the potential for severe birth defects, it is important to confirm the client is not pregnant. "You will need to take a vitamin A supplement twice daily.": Vitamin A enhances the risk of isotretinoin toxicity. The client should avoid taking vitamin A supplements because isotretinoin is a derivative of vitamin A.

Once month later, the client tells the provider that she has developed some adverse effects, Which of the following eff3ects does clomiphene cause? (Select all that apply.) A. Blurred vision B. Hot flashes C. Breast engorgement D. Muscle pain E. Nausea

A, B, C, E A. Blurred vision B. Hot flashes C. Breast engorgement E. Nausea Clomiphene can cause visual disturbances. The client should avoid driving and engaging in activities that may be dangerous, and if the symptoms persist, she should have an ophthalmology examination. For severe vision changes, she should stop taking the drug. Clomiphene may cause hot flashes and breast engorgement. It might help if she wears loose-fitting clothing, applies cold compresses to her breasts, and wears a supportive bra. Nausea and vomiting may develop with clomiphene therapy. If they do, she could try taking the drug with food to minimize these effects. Clomiphene is more likely to cause abdominal pain than muscle pain. (The Reproductive and Genitourinary Systems Case Study)

Primary care providers prescribe drug dosages for children using which of the following information? (Select all that apply.) A. Body surface area B. Age C. Weight D. Developmental level E. Drug properties

A, B, C, E A. Body surface area B. Age C. Weight E. Drug properties To avoid toxicity, overdose, or undertreatment, primary care providers adjust drugs for children based on their body surface area, age, and weight, along with drug properties such as metabolism and excretion. The childs developmental level may determine the method of administration but not the appropriate dosage. (Introduction to Pharmacology Activity )

You are caring for a client who is taking oral amiodarone to treat fibrillation. Recognizing the adverse effects of amiodarone, you advise the client to watch for and report which of the following? (Select all that apply.) A. Cough B. Blurred vision C. Dizziness D. Sore throat E. Fever

A, B, C, E A. Cough B. Blurred vision C. Dizziness E. Fever Amiodarone, a potassium channel blocker, can cause pulmonary toxicity, manifesting as dyspnea, cough, fever, and chest pain. The drug can also cause corneal microdeposits, causing blurred vision and photophobia. Other possible adverse effects include dizziness and ataxia. It is unlikely to cause a sore throat; however, it can cause a dry cough, due to Heart failure or ARDS. (The Cardiovascular System Activity)

A client is about to start taking hydrocortisone (Cortef) to treat adrenocortical insufficiency. You should instruct the client to do which of the following to help reduce the risk for adverse effects of this drug? (Select all that apply.) A. Increase her calcium and vitamin D intake B. Take the drug with food C. Record weigh regularly D. Urinate every 4 hr E. Report increased stress

A, B, C, E A. Increase her calcium and vitamin D intake B. Take the drug with food C. Record weigh regularly E. Report increased stress Hydrocortisone can cause bone loss. Instruct the client to increase intake of calcium and vitamin D and increase weight-bearing activity. Hydrocortisone can cause peptic ulcer disease and gastric distress. Suggest taking the drug with food and avoid NSAIDs, especially aspirin. Hydrocortisone can cause fluid and electrolyte imbalances, such as hypernatremia. Advise the client to weigh regularly and report weight gain or edema. Hydrocortisone is unlikely to cause urinary retention, although it can cause urinary urgency and frequency. Tell the client to report increased stress, as dosage might require an adjustment during stressful times. (The Endocrine System Activity)

A nurse is caring for a client who is receiving methylprednisolone. Which of the following laboratory values should the nurse plan to monitor? (Select all that apply.) A. White blood cell count B. Serum potassium C. Creatine phosphokinase D. Blood glucose E. Amylase

A, B, D White blood cell count: Methylprednisolone can increase the clients risk for infection and cause leukocytosis. Serum potassium: Methylprednisolone can cause hypokalemia, as well as fluid and sodium retention. Blood glucose: Methylprednisolone can cause increased blood glucose levels. Creatine phosphokinase: Methylprednisolone does not damage the muscles and, therefore, does not cause release of creatine phosphokinase. Amylase: Methylprednisolone does not affect pancreatic function.

The health care provider administers oxytocin for induction of labor. The health care provider should monitor which of the following in order to assess for an adverse reaction of oxytocin? (Select all that apply.) A. Urine output B. Blood pressure C. Level of consciousness D. Duration of contractions E. Fetal heart rate

A, B, D, E A. Urine output B. Blood pressure D. Duration of contractions E. Fetal heart rate Oxytocin, a uterine stimulant, may cause water retention. Monitor intake and output and indications of water intoxication, such as drowsiness or headache. Oxytocin may cause hypertensive crisis. Monitor blood pressure and fetal and maternal heart rate. Oxytocin may cause uterine hyper stimulation and fetal distress. Monitor length, strength, and duration of contractions. Stop infusion, turn client to side and report contractions lasting greater than 60 seconds. Oxytocin is unlikely to affect level of consciousness, but it can cause anxiety. (The Reproductive and Genitourinary Systems Activity)

A nurse is reinforcing discharge teaching with a client who has type 2 diabetes mellitus and the new prescription for metformin. The nurse should instruct the client to monitor for which of the following manifestations as adverse effects of this medication? (Select all that apply.) A. Muscle pain B. Fatigue C. Weight gain D. Constipation E. Metallic taste

A, B, E A. Muscle pain: Clients taking metformin should monitor for muscle pain or hyperventilation as indication of lactic acidosis. B. Fatigue: Metformin can decrease vitamin B12 absorption, causing pernicious anemia. Therefore, clients taking metformin should monitor for manifestations of anemia, including fatigue and pallor. Absence of adequate vitamin B12 can also cause neurological damage. E. Metallic taste: Metformin can cause GI manifestations such as bitter or metallic taste. Weight gain: Unlike sulfonylurea medications, metformin does not cause weight gain, and can cause weight loss in some clients. Constipation: Metformin can cause GI manifestations such as diarrhea, nausea, and vomiting.

A client is starting therapy with psyllium. You explain to the client that psyllium will have which of the following therapeutic effects? (Select all that apply.) A. Relieve constipation B. Increase gastric pH C. Reduce diarrhea D. Decrease gastric acid secretion E. Alleviate nausea

A, C A. Relieve constipation C. Reduce diarrhea Psyllium, a bulk laxative, treats constipation and reduces diarrhea by increasing bulk to create soft, formed stools. It helps clients who have diverticulosis, irritable bowel syndrome, and fecal ostomies. Aluminum hydroxide, an antacid, increased gastric pH. Omeprazole, a proton pump inhibitor, decreases the secretion of gastric acid. Ondansetron, a serotonin antagonist, helps relieve nausea and vomiting. (The Gastrointestinal System Activity)

The provider discharges the client to home with a prescription for warfarin. You should advise the client to do which of the following? (Select all that apply.) A. Avoid taking NSAIDs B. Use a disposable razor C. Brush teeth with a soft toothbrush D. Increase intake of dark-green, leafy vegetables E. Ask the provider before taking over-the-counter drugs

A, C, E A. Avoid taking NSAIDs C. Brush teeth with a soft toothbrush E. Ask the provider before taking over-the-counter drugs You should instruct the client to avoid taking NSAIDs and other over-the-counter drugs that could interact with warfarin. NSAIDs, especially aspirin, can increase anticoagulation and cause bleeding. Many other drugs interact with warfarin, so it is essential for the client to check with the provider before taking them. The client should use an electric razor to prevent injuries that might bleed and should brush with a soft toothbrush to avoid gum irritation and bleeding. Dark-green, leafy vegetables are high in vitamin K and, if consumed in excessive amounts, can reduce anticoagulation. Clients should monitor and report bleeding, such as excessive or easy bruising and blood in the urine, stools, or emesis. (The Hematologic System Case Study)

Digoxin is effective in treating heart failure due to which pharmacological actions? (Select all that apply.) A. Exerts positive inotropic action B. Decreases cardiac output C. Decreases sympathetic tone D. Increases vascular constriction E. Increased myocardial contractility

A, C, E A. Exerts positive inotropic action C. Decreases sympathetic tone E. Increases myocardial contractility Digoxin, a cardiac glycoside, exerts positive inotropic action on the cardiovascular system. As a result, it increases strong volume and cardiac output by increased myocardial contractility, which, in turn, increases the force of ventricular contraction. This drug also reduces sympathetic tone, slows the heart rate, and decreases venous pressure by decreasing vasoconstriction. These effects result in a increased renal blood flow, thus promoting urine production and excretion. That reduction in retained fluid decreases blood volume and relieves pulmonary congestion and peripheral edema. (The Cardiovascular System Case Study)

The health care professional is providing instructions to the client on taking aluminum hydroxide. Which information should be included? (Select all that apply.) A. Check labels for sodium and phosphate content B. Decrease activity and exercise C. Increase fluid and fiber intake D. Report whitish colored stools E. Monitor for signs of constipation

A, C, E A. Check labels for sodium and phosphate content C. Increase fluid and fiber intake E. Monitor for signs of constipation Some formulations of aluminum hydroxide, an antacid, can contain significant amounts of sodium, which can cause client blood sodium levels to increase. This is particularly harmful for clients who have hypertension. The healthcare professional should instruct the client to monitor sodium intake. Aluminum hydroxide can cause constipation, so the client should increase her fluid and fiber intake and increase exercise and activity. The client should expect whitish or white-speckled stools but should report bloody or black, tarry stools, as they indicate gastrointestinal bleeding. (The Gastrointestinal System Case Study)

A primary care provider prescribes sumatriptan to treat a clients migraine headaches. Which instructions should the nurse include when telling the client how to take sumatriptan? (Select all that apply.) A. Take the drug after onset of the migraine. B. Administer one nasal spray into each nostril. C. Take it once per day to prevent migraine. D. Repeat the dose once if the headache returns. E. Allow the tablet to dissolve under the tongue.

A, D A. Take the drug after onset of the migraine. D. Repeat the dose once if the headache returns. Clients should only take sumatriptan, a serotonin agonist, after the onset of a migraine or a cluster headache. They should spray the drug once into one nostril. They may repeat the dose once, 1 to 2 hours after the initial dose, depending on the route. They should not use sumatriptan sublingually; only oral, subcutaneous, and intranasal routes are appropriate. (The Neurological System Activity (Part 1))

A client is about to start taking clopidogrel to prevent transient ischemic attacks. You should instruct the client to watch for and report which of the following adverse effects of this drug? (Select all that apply.) A. Petechiae B. Tinnitus C. Diaphoresis D. Weakness E. Vision changes

A, D, E A. Petechiae D. Weakness E. Vision changes Clopidogrel, an antiplatelet drug, can cause bleeding. Tell the client to watch for and report any indication for any indications of bleeding, such as petechiae, bruising, bleeding gums, abdominal pain, or hematemesis. Monitor for indications of a hemorrhagic stroke, such as severe headache, unilateral weakness, disturbed vision, or aphasia. Clopidogrel is unlikely to cause tinnitus or diaphoresis. Aspirin is an example of a drug that can cause tinnitus and diaphoresis, especially with excessive doses. (The Hematologic System Activity)

A nurse in a community health clinic is preparing to administer the varicella vaccine to a young adult female client who has not previously had chickenpox or its vaccine. The nurse should withhold the vaccine and collect additional data when the client makes which of the following statements? A. "I am allergic to neomycin." B. "I am taking antibiotics for my acne." C. "My irritable bowel syndrome has been acting up the last few days." D. "I have been taking an oral contraceptive for the last 6 months."

A. "I am allergic to neomycin.": A hypersensitivity to neomycin is a contraindication for receiving the varicella vaccine. "I am taking antibiotics for my acne.": Antibiotics are not a contraindication for receiving the varicella vaccine. "My irritable bowel syndrome has been acting up the last few days.": Irritable bowel syndrome is not contraindication for receiving the varicella vaccine. "I have been taking an oral contraceptive for the last 6 months.": Taking oral contraceptives is not a contraindication for receiving the varicella vaccine. The nurse should recognize that pregnancy is a contraindication for receiving the varicella vaccine.

A nurse on a medical-surgical unit is preparing to administer medications to a client. Which of the following questions should the nurse ask the client to verify the clients identity? A. "What is your name and date of birth?" B. "What is your room number and bed number?" C. "What is your doctors name?" D. "What is your diagnosis?"

A. "What is your name and date of birth?": Acceptable client identifiers include the clients name, telephone number, facility identification number, date of birth, and other client-specific identifiers. The nurse must use at least two identifiers to verify the clients identity and should compare the information to what is the clients wristband or the medical record. "What is your room number and bed number?": Room number and bed number are not reliable indicators of a clients identity because the client might have been transferred to another room or bed. "What is your doctors name?": Doctors name is not a reliable indicator of a clients identity because multiple clients have the same doctor or provider. The nurse must use at least two client-specific identifiers to verify the clients identity. "What is your diagnosis?": Diagnosis is not a reliable indicator of a clients identity because multiple clients can have the same diagnosis or disorder. The nurse must use at least two client-specific identifiers to very the clients identity.

A nurse is reinforcing teaching with a client who is to start therapy with a nitroglycerin transdermal patch. Which of the following statements by the client indicates an understanding of the teaching? A. "While using the patch, I will be careful when rising from a chair." B. "I should leave the patch in place for 24 hours." C. "I should apply the patch to the same location with each application." D. "I will apply a patch if i have chest pain."

A. "While using the patch, I will be careful when rising from a chair.": Nitroglycerin can cause orthostatic hypotension, which can result in dizziness. The client should change positions slowly to avoid injury. "I should leave the patch in place for 24 hours.": The client should apply the patch each morning but should remove it after 12 to 14 hr and allow for 10 to 12 hr without any medication. This will help prevent the development of tolerance to the medication. "I should apply the patch to the same location with each application.": The client should rotate application sites to avoid skin irritation. "I will apply a patch if i have chest pain.": The client should use rapid-onset, short-acting nitroglycerin to treat acute angina. The nitroglycerin transdermal patch is a sustained-release medication and is not used to treat chest pain.

A nurse is planning to reinforce teaching regarding newborn immunizations with a client who is 24 hr postpartum. which of the following information should the nurse plan include? A. "Your baby will receive the first hepatitis B vaccine before discharge." B. "Your baby will receive the rotavirus vaccine if your blood titer is low." C. "Your baby will receive the first influenza virus vaccine at the 4-week checkup." D. "Your baby will receive the varicella vaccine if you have a history of chickenpox."

A. "Your baby will receive the first hepatitis B vaccine before discharge.": The newborn should receive the first hepatitis B vaccine at birth with the next two doses at least 2 months apart. "Your baby will receive the rotavirus vaccine if your blood titer is low.": The blood titer of a newborns mother does not determine the need for a rotavirus vaccine. The rotavirus vaccine is administered every 2 months starting at 24 weeks. "Your baby will receive the first influenza virus vaccine at the 4-week checkup.": Children should receive an annual influenza virus vaccine starting at age 6 months. "Your baby will receive the varicella vaccine if you have history of chickenpox.": Children should receive the first of two doses of the varicella vaccine between 12 and 15 months.

You are instructing a client about using cromolyn via inhalation to treat exercise-induced bronchospasm. You advise the client to take the drug. A. 15 minutes before exercising B. If needed to stop bronchospasm C. After strenuous activity D. Once in the evening at bedtime

A. 15 minutes before exercising Cromolyn may prevent exercise-induced asthma if used 15 minutes before exercising. Cromolyn is not effective at stopping bronchospasm. Clients may use cromolyn via nebulizer up to 4 times per day to prevent bronchospasm. Using the inhaler at bedtime will not prevent exercise-induced asthma. (The Respiratory System Activity)

You should instruct a client who is taking zileuton to report which of the following possible indications of a serious adverse effect of this drug? A. Abdominal pain B. Dysphagia C. Blurred vision D. Bradycardia

A. Abdominal pain Zileuton may cause liver damage and hepatitis. Clients should report any signs of hepatic toxicity, such as abdominal pain or jaundice. Dysphagia, or difficulty swallowing, is not likely to result from taking zileuton, although this drug can cause neck pain and rigidity. Zileuton is unlikely to cause blurred vision, although it can cause conjunctivitis. Bradycardia is unlikely to result from taking zileuton, although this drug can cause chest pain. (The Respiratory System Activity)

A nurse is reinforcing teaching with a client who has seizures and a new prescription for valproic acid. The nurse should instruct the client to report which of the following adverse effects of valproic acid immediately to the provider? A. Abdominal pain B. Hair loss C. Weight gain D. Ataxia

A. Abdominal pain: The greatest risk to the client is hepatotoxicity and pancreatitis, which can cause abdominal pain. Therefore, the client should notify the provider immediately if experiencing a decrease in appetite, abdominal pain, or yellowing of the skin. Hair loss: The client is at risk for hair loss because it is an adverse effect of taking valproic acid. However, another adverse effect is the priority to report to the provider. Weight gain: The client is at risk for weight gain because it is an adverse effect of taking valproic acid. However, another adverse effect is the priority to report to the provider. Ataxia: The client is at risk for ataxia because it is an adverse effect of taking valproic acid. However, another adverse effect is the priority to report to the provider.

A client is about to start taking alosetron to treat diarrhea. you should instruct the clients to watch for and report which of the following adverse drug reactions of this drug? A. Constipation B. Difficulty swallowing C. Blurred vision D. Confusion

A. Constipation Alosetron, a serotonin 5-HT3 receptor antagonist, can cause constipation that can lead to impaction and perforation. The client should monitor bowel patterns and stop the drug for constipation. Alosetron is unlikely to cause difficulty swallowing. Psyllium, a fiber supplement, can cause esophageal obstruction and difficulty swallowing. instructs clients to take psyllium with 8 oz of water. Alosetron is unlikely to cause blurred vision. Dimenhydrinate, an antihistamine, can cause blurred vision. Alosetron is unlikely to cause confusion. (The Gastrointestinal System Activity)

A nurse is caring for a client who has multiple sclerosis and has new prescription for baclofen. Which of the following findings indicates to the nurse that the medication is having a therapeutic effect? A. Decreased muscle spasticity B. Increased urinary output C. Increased mental alertness D. Decreased heart rate

A. Decreased muscle spasticity: The nurse should identify that baclofen is an antispasmodic that decreases muscle spasticity in a client who has multiple sclerosis. Increased urinary output: Urinary frequency is an adverse effect of baclofen rather than a therapeutic effect. Increased mental alertness: Baclofen is an antispasmodic that can cause CNS adverse effects, such as drowsiness, fatigue, and confusion, and does not produce an increase in the clients mental alertness as therapeutic effect. Decreased heart rate: baclofen is an antispasmodic and does not decrease the clients heart rate as a therapeutic effect.

Combination oral contraceptives, such as ethinyl estradiol and drospirenone, should be used with caution in clients with which of the following disorders? A. Diabetes B. Hypothyroidism C. Thrombocytopenia D. Osteoporosis

A. Diabetes Combination oral contraceptives, such as ethinyl estradiol and drospirenone may cause glucose intolerance. Clients with diabetes may need to increase the dosage of insulin or diabetic agent. Monitor serum glucose carefully. Combination oral contraceptives are unlikely to adversely affect clients with hypothyroidism. Clients with an increased risk for clotting, such as a history of thrombophleitis, should not use oral contraceptives. Combination oral contraceptives are unlikely to adversely affect clients with thrombocytopenia. Clients with osteoporosis may take combination oral contraceptives. (The Reproductive and Genitourinary Systems Activity)

A primary care provider prescribes captopril to treat the clients hypertension and also help alleviate his heart failure. It is essential for a health care provider to make sure a client understands that he should not take which over-the-counter drugs while taking captorpil? A. Ibuprofen (Advil, Motrin) B. Acetaminophen (Tylenol) C. Calcium Carbonate (Tums) D. Diphenhydramine (Benadryl)

A. Ibuprofen (Advil, Motrin) Taking a nonsteroidal anti-inflammatory drug (NSAID), such as ibuprofen or aspirin, concurrently with captopril, an angiotensin-converting enzyme (ACE) inhibitor, reduces the ACE inhibitors antihypertensive effects. Acetaminophen, calcium carbonate, and diphenhydramine do not interact adversely with captopril; however, healthcare professionals should make sure clients understand that food decreases the absorption of captopril. (The Cardiovascular System Case Study)

A nurse is reinforcing teaching with a client who has a new prescription for etanercept to treat rheumatoid arthritis. Which of the following instructions about self-administering this medication should nurse include. A. Discard any solutions that are cloudy. B. Attach a 21-gauge needle to the syringe for injection. C. Self-administer the medication on alternate days. D. Shake the reconstituted solution well before self-administration.

A. Discard any solutions that are cloudy: The client should discard any vitals or pre-filled syringes that contain solutions that are discolored, cloudy, or have any sediment in them. Attach a 21-gauge needle to the syringe for injection: The client should attach a 27-guage need to the syringe for injecting the medication subcutaneously. Self administer the medication on alternate days: The client should self-administer the medication once per week. Shake the reconstituted solution well before self-administration: The client should swirl the solution gently before self-administration.

You should recognize that which of the following is a contraindication for taking codeine? A. Emphysema B. Cataracts C. Hyperthyroidism D. Nonproductive cough

A. Emphysema Codeine is unsafe for clients who have decreased respiratory reserve, such as with emphysema or asthma. Cataracts are not a contraindication for codeine use, although the drug can exacerbate some types of glaucoma due to a possible increase in intracranial pressure. Codeine may cause constipation, hypotension and reduced respiration. Cautious use is essential for clients with hypothyroidism because the drug can worsen bradycardia codeine can help suppress a nonproductive cough. (The Respiratory System Activity)

A nurse is reviewing the medication administration record for a client who has a new prescription for tobramycin to treat pulmonary infection. Which of the following medications should the nurse identify as increasing the risk for ototoxicity while taking tobramycin? A. Furosemide B. Propranolol C. Gabapentin D. Guaifenesin

A. Furosemide: Tobramycin is an aminoglycoside antibiotic that can cause ototoxicity. Furosemide is a diuretic that also can cause ototoxicity. The clients risk for hearing loss is increased if receiving both of these medications at the same time. Propranolol: Propranolol is a beta blocker that does not interact with tobramycin. Gabapentin: Gabapentin is an anticonvulsant that does not interact with tobramycin. Guaifenesin: Guaifenesin is an expectorant that does not interact with tobramycin.

A nurse is collecting data from the parent of a toddler who is about to receive the varicella immunization. The nurse should identify that an anaphylactic reaction to which of the following substances is contraindication for receiving this immunization? A. Gelatin B. Penicillin C. Sulfa D. Eggs

A. Gelatin: The nurse should identify that hypersensitivity reactions to either gelatin or neomycin are contraindications for receiving the varicella vaccine because it contains both of theses substances. Penicillin: An anaphylactic reaction to penicillin is not a contraindication to receiving this vaccine. Anaphylactic reactions to specific immunizations or to their components are true contraindications for receiving those vaccines. Sulfa: A hypersensitivity reaction to sulfa is a contraindication for receiving medications containing sulfonamide. Eggs: Administering the measles, mumps, and rubella vaccine requires caution with clients who have had the hypersensitivity reactions to eggs, gelatin, or neomycin.

You are caring for a client who is preoperative and is NPO. The client takes carbamazepine, 200 mg PO BID for a seizure disorder. Which of the action should the nurse take? A. Give the oral dose of carbamazepine with a sip of water. B. Give carbamazepine IV. C. Withhold the carbamazepine. D. Administer half the usual morning dose of carbamazepine.

A. Give the oral dose of carbamazepine with a sip of water. It is important to continue the oral carbamazepine to maintain therapeutic blood levels and to prevent seizure activity. You should notify the anesthesiologist and should give the client the carbamazepine with a sip of water. Carbamazepine is not available IV. Withholding the carbamazepine may reduce the therapeutic blood level and cause seizure activity. Administering a reduced dose of carbamazepine may reduce the therapeutic blood level and cause seizure activity. (The Neurological System Activity (Part 1))

You are caring for a client who is taking levothyroxine (Synthroid) to treat hypothyroidism. The client reports palpitations, weight loss, and diarrhea. You suspect which of the following adverse effects of this drug? A. Hyperthyroidism B. Addison's disease C. Myxedema D. Hyperglycemia

A. Hyperthyroidism Levothyroxine, a thyroid hormone replacement drug, can cause hyperthyroidism if the prescribed dose exceeds what the client require to remain euthyroid. Monitor thyroid function via T4 and TSH levels. Instruct the client to watch for and report induction of hyperthyroidism, such as anxiety , insomnia, tachycardia, palpitations, diarrhea, and weight loss. Levothyroxine does not cause myxedema or Addison's disease. Addison's disease is due to adrenal insufficiency, and levothyroxine treats myxedema coma. Levothyroxine is unlikely to cause hyperglycemia. Injectable hypoglycemics, such as insulin, can cause hypoglycemia. (The Endocrine System Activity)

A client is taking bumetanide and spironolactone. You explain to the client that using these drugs together may reduce the risk of which adverse effects? A. Hypokalemia B. Hyponatremia C. Hyperglycemia D. Hypochloremia

A. Hypokalemia Spironolactone, a potassium-sparing diuretic, causes retention of potassium. This reduces the risk of hypokalemia due to potassium loss from bumetanide, a high-ceiling loop diuretic. Using these two drugs together will not affect sodium, glucose, or chloride levels. Hyperglycemia, hyponatremia, and hypochloremia remain risks with the use of bumetanide. Hyponatremia remains a risk with the use of spironolactone. (The Cardiovascular System Activity)

As the client continues to stabilize, she is prescribed prednisone by mouth. You review her history and the drugs she has been taking. Which of the following drugs might interact adversely with prednisone? A. Ibuprofen B. Levothyroxine C. Atorvastatin D. Spironolactone

A. Ibuprofen Nonsteroidal anti-inflammatory drugs, such as ibuprofen, can increase the risk of gastrointestinal ulceration and should be avoided. levothyroxine, a thyroid hormone, should not interact with prednisone. This drug does, however, interact with oral anticoagulants. Atorvastatin, an HMG-CoA reductase inhibitor, should not interact with the prednisone. This drugs does, however interact with macrolide antibiotics. Potassium-sparing diuretics, such as spironolactone, should not interact with prednisone. This drug does, however interact with cardiac glycosides. Diuretics that can cause potassium loss should be avoided. (The Respiratory System Case Study)

A nurse is caring for a client who has schizophrenia and is to start therapy with risperidone. For which of the following manifestations should the nurse monitor to determine whether the treatment is effective? A. Improved social interactions B. Decreased obsessive-compulsive thoughts C. Decreased hand tremors D. Improved urinary control

A. Improved social interactions: Clients who have schizophrenia typically have difficulty interacting with others and maintaining relationships. Manifestations can include dull affect and speech deficiency. Risperidone is atypical antipsychotic that can minimize theses manifestations, improving social interactions with others. Decreased obsessive-compulsive thoughts: risperidone does not decreases obsessives-compulsive thoughts. Clients who have obsessives-compulsive disorder may take an SSRI, such as fluoxetine, to decrease obsessives-compulsive thoughts. Decreased hand tremors: Risperidone does not decrease hand tremors. Clients who have bipolar disorder may take beta-adrenergic blocking agent, such as propranolol, to decrease hand tremors. Improved urinary control: risperidone does not improve urinary control. Clients who have an overactive bladder may take oxybutynin to reduce urinary urgency and frequency, nocturia, and urge incontinence.

A nurse is caring for a client taking etanercept. Which of the following adverse effects is most important to mention when educating this client? A. Injection site reaction can occur. B. Elevation of liver enzymes can occur. C. Gastric perforation can occur. D. Vision changes can occur.

A. Injection site reaction can occur. Injection site reaction occurs in 37% of clients taking etanercept. Teach the client to look for localized reaction signs, such as itching, erythema, swelling, and pain. Elevation of liver enzymes is an adverse effect of the drug leflunomide. Gastrointestinal perforation has been associated with the drug methotrexate. Change in vision is the adverse effect of alendronate. (The Musculoskeletal System Activity)

A client is about to start taking raloxifene to treat osteoporosis. You advise the client to watch for which of the following adverse effects? A. Hot flashes B. Breast cancer C. Blurred vision D. Jaw pain

A. Jaw pain Raloxifene, a selective estrogen receptor modulator that helps reduce the risk of breast cancer, may cause hot flashes, leg cramps, and pulmonary embolism. Bisphophonates can cause jaw pain and blurred vision. (The Musculoskeletal System Activity)

A nurse is reinforcing teaching with a client about the adverse effects of simvastatin. For which of the following adverse effects should the nurse instruct the client to notify the provider? A. Muscle pain B. Fine hand tremors C. Urinary retention D. Double vision

A. Muscle pain: The nurse should instruct the client to notify the provider if muscle pain or tenderness develops because this can indicate the client is developing rhabdomyolysis. Fine hand tremors: Fine hand tremors are not a adverse effect of simvastatin. Urinary retention: Urinary retention are not a adverse effect of simvastatin. Double vision: Double vision are not a adverse effect of simvastatin.

The health care professional is now assessing the client. Which of the following findings can be a early indication of phenytoin toxicity? A. Nystagmus B. Tachycardia C. Tachypnea D. Jaundice

A. Nystagmus Phenytoin toxicity can develop with serum phenytoin levels greater than 20 mcg/mL, causing nystagmus, ataxia, sedation, and blurred or double vision. Rapid infusion of phenytoin can cause cardiac dysrhythmias, bradycardia, and hypotension. Toxic levels of phenobarbital, sometimes given phenytoin, also can cause bradycardia. Respiratory depression can be a sign of phenytoin or phenobarbital toxicity. Valproic acid can cause hepatotoxicity, which often manifests as jaundice. (The Neurological System Case Study (Part 1))

A client is about to start taking cyanocobalamin, a vitamin B12 preparation. You understand that this drug treats which of the following types of anemia? A. Pernicious B. Sickle cell C. Aplastic D. Iron deficiency

A. Pernicious Cyanocoblamin, a vitamin B12 preparation, treats pernicious anemia. This type of anemia is a vitamin B12 deficiency due to the lack of intrinsic factor, which the body needs to absorb vitamin B12. Cyanocobalamin also treats vitamin B12 deficiency from gastric surgery or enteritis. Sickle cell anemia is a genetic disease with which RBCs are crescent-shaped and reduced in number. The sickle cells do not carry oxygen well, and they die prematurely. Apalstic anemia is a disorder from too few stem cells, which the body needs to produce new blood cells. In iron deficiency anemia, too little iron results in too few red blood cells. (The Hematologic System Activity)

Which of the following drugs are effective in disease prevention as well as treatment of osteoporosis? (Select all that apply.) A. Raloxifene B. Calcitonin-salmon C. Alendronate D. NSAIDs

A. Raloxifene C. Alendronate The Selective Estrogen Receptor Modulator (SERM) raloxifene is used in both the prevention and treatment of postmenopausal osteoporosis. Alendronate, a bisphosphonate, is used in the prevention and treatment of postmenopausal osteoporosis, age-related osteoporosis in men, and glucocorticoid-related osteoporosis in clients taking glucocorticoids long term. Calcitonin-salmon is used in the treatment, not prevention, of established postmenopausal osteoporosis. NSAIDs treat symptoms of a disease and are not effective in prevention. (The Musculoskeletal System Activity)

An older client may be at risk for drug toxicity dur to which of the following physiologic changes associated with aging? A. Reduced hepatic blood flow B. Increased glomerular filtration rate C. Reduced body fat D. Increased gastric motility

A. Reduced hepatic blood flow Older clients may be at risk for drug toxicity due to delayed drug metabolism related to reduced hepatic blood flow and decreased liver enzyme production. Other physiologic changes in older adults are decreased blood flow to the kidneys and reduced glomerular filtration rate, increased body fat, decreased cardiac output, decreased gastric emptying, and slowed peristalsis. (Introduction to Pharmacology Activity)

A client with glaucoma asks you about taking oxybutynin, a muscarinic antagonist, to mange an overactive bladder. you explain that glaucoma is a contraindication for taking oxybutynin. Primary care providers should not prescribe contraindicated drugs because of their potential for which of the following? A. Serious adverse reactions B. Drug tolerance C. Drug dependence D. Interactions with other drugs

A. Serious adverse reactions Some drugs are unsafe for clients with certain characteristics or disorders or because they eat certain foods or take certain drugs as they have the potential to cause serious or life-threatening adverse effects. Drug tolerance is a decreased response to a drug over a period of time. Drug dependence is a physiologic or psychologic need for a drug. A drug interaction is an alteration in the expected pharmacologic action of a drug that results from an interaction with another drug. (Introduction into Pharmacology Activity)

The health care provider instructs the client about the importance of taking the prescribed 1,500 mg/day dosage of calcium carbonate (Tums). Which of the following instructions should the health care provider recommend? A. Set up a simple reminder system. B. Take the three tablets at the same time. C. Reduce fluid intake to increase effectiveness. D. Take the drug with meals to reduce gastric irritation.

A. Set up a simple reminder system. If the clients nonadherence is unintentional, the health care provider can work with her to devise a reminder system that will work for her, such as a calendar, a tablet counter, or a watch alarm routine. Clients should take no more than 600 mg of calcium at one time for optimal absorption. Clients should take calcium carbonate that is supplied in a chewable form such as Tums on an empty stomach followed by a full glass of water. Calcium carbonate is also used as an antacid to neutralize gastric acid. It is not necessary to take it with meals to reduce gastric irritation. (The Musculoskeletal System Case Study)

A nurse is collecting data from a client who has hyperthyroidism and a new prescription for propylthiouracil. The nurse should monitor the client for which of the following manifestations as an adverse effect of this medication? A. Sore throat B. Metallic taste C. Mania D. Urinary retention

A. Sore throat: The nurse should monitor for sore throat and fever because these are early indications of agranulocytosis, which is an adverse affect of propylthiouracil. Metallic taste: The nurse should monitor for loss of taste, not metallic taste, as an adverse effect of propylthiouracil. Mania: The nurse should monitor for drowsiness, not mania, as an adverse effect of propylthiouracil. Urinary retention: The nurse should monitor for glomerulonephritis, not urinary retention, as an adverse effect of propylthiouracil.

A nurse is collecting data from a client who recently began taking niacin to treat hyperlipidemia. The client reports experiencing facial flushing after taking the medication. The nurse should expect the provider to make which of the following recommendations? A. Take aspirin 30 min before taking the medication. B. Lie down for 30 min after taking the medication. C. Take the medication on an empty stomach. D. Avoid caffeine while takin the medication.

A. Take aspirin 30 min before taking the medication.: The nurse should expect a recommendation of aspirin 30 min prior to each dose to minimize facial flushing. The nurse should inform the client that this manifestation is often transient and diminishes over time. Lie down for 30 after taking the medication.: The nurse should instruct the client to change positions slowly while taking this medication to avoid orthostatic hypotension. However, lying down will not reduce facial flushing. Take the medication on an empty stomach.: The nurse should instruct the client to take this medication with meals or milk to reduce gastrointestinal irritation. However, taking the medication on an empty stomach will not reduce facial flushing. Avoid caffeine while taking the medication.: The nurse should instruct the client to minimize intake of fat and cholesterol to reduce hyperlipidemia. However, avoiding caffeine will not reduce facial flushing.

At a visit to the providers office, an older adult clients partner reports that the client has nausea, diarrhea, and a loss of appetite. He has a history of Alzheimer's disease and is taking donepezil. Which information should be provided by the nurse? A. Take the drug with food B. Stop taking the drug right away C. Take a reduced dosage of the drug D. undergo liver function tests right away

A. Take the drug with food Cholinesterase inhibitors, such as donepezil, commonly cause nausea, vomiting, anorexia, and other gastrointestinal symptoms. This is due to the increase of acetylcholine in all parts of the body and its subsequent effect on the gastrointestinal tract and other system of the body. The provider should explain to the client and his caregiver that taking the drug with food and at bedtime will help minimize uncomfortable gastrointestinal effects. The provider also should instruct the partner to report vomiting, severe diarrhea, weight loss, or signs of gastrointestinal bleeding, such as dark stools or coffee-ground emesis, especially if the client is also taking a nonsteroidal anti-inflammatory drug. At this point, it is not necessary to stop taking the drug, reduce the dosage, or undergo liver function tests. (The Neurological System Activity (Part 1))

The health care provider explains to the client that there may be a drug interaction between the glucocorticoid and the calcium supplement. Which of the following may occur? A. The glucocorticoid may cause reduced absorption of the calcium supplement. B. The glucocorticoid may potentiate the effects of the calcium supplement. C. The oral calcium supplement used with the glucocorticoid may cause bradycardia. D. The calcium supplement may increase the risk for peptic ulcer disease.

A. The glucocorticoid may cause reduced absorption of the calcium supplement. Because the glucocorticoid can reduce the absorption of calcium, clients should take the calcium supplement one hour before or one to two hours after they take the glucocorticoid. Glucocorticoids can increase potassium loss, but they are unlikely to affect the calcium levels without supplements. Parenteral calcium given to clients who are on digoxin can cause severe bradycardia. The calcium supplement should not increase the risk for peptic ulcer disease; however, it can increase the risk for renal calculi (kidney stones). (The Musculoskeletal System Case Study)

The health care professional is talking with the client, who is soon to be discharged to home, about the adverse effects of phenytoin. Which of the following instructions may help minimize the adverse effects of phenytoin? A. Use good oral hygiene. B. Take the drug on an empty stomach. C. Use sunblock when outdoors. D. Decrease dietary fiber.

A. Use good oral hygiene. The risk of gingival hyperplasia may be reduced with daily flossing and gum massage. Gastric upset my be reduced by taking phenytoin with food. Phenytoin is unlikely to cause photosensitivity, but it can cause photophobia. It is unlikely to cause diarrhea, but it can cause constipation, so increased fiber intake might help minimize or prevent that effect. (The Neurological System Case Study (Part 1))

While the client is still receiving continuous IV heparin (enoxaparin), the provider prescribes oral warfarin. This is because A. Warfarin takes 3 to 5 days to achieve therapeutic effects B. IV heparin alone becomes ineffective after the first 1 to 2 days C. Abrupt cessation of heparin therapy increases the risk for thrombocytopenia D. Warfarin increases the risk for hemorrhage

A. Warfarin takes 2 to 5 days to achieve therapeutic effects Warfarin takes 3 to 5 days to achieve its therapeutic effects. Once it provides effective anticoagulation, with an INR of 2 to 3, the provider should discontinue heparin therapy. IV heparin is effective as an anticoagulant but becomes impractical and costly for the client to receive at home and can increase the risk of bleeding. Stopping heparin therapy does not increase the risk for thrombocytopenia. It actually decreases the risk in this case, because thrombocytopenia is not an adverse effect of warfarin. Similar to heparin, however, warfarin can cause bleeding. (The Hematologic System Case Study)

You should advise a client who is taking beclomethasone to watch for which of the following possible indications of an adverse effect of this drug? A. White coating in the mouth B. Abdominal pain C. Muscle twitching D. Palpitations

A. Which coating in the mouth Beclomethasone, an inhaled glucocorticoid, may cause oropharyngeal candidiasis. Clients should gargle after each use, use a spacer to reduce the amount of drug in the mouth and throat, and report any white patches inside the mouth or on the tongue. Leukotriene modifiers are more likely to cause liver injury and abdominal pain. Muscle twitching and palpitations may indicate methylxanthine toxicity. (The Respiratory System Activity)

The following day, the client is stable and being prepared for discharge. You explain that she will continue to take prednisone orally at home, as prescribed, and that she should watch for and report which of the following to her usual provider? (Select all that apply.) A. Persistent cough B. Black, tarry tools C. Sore throat D. Muscle weakness E. Urinary retention

B, C, D B. Black, tarry stools C. Sore throat D. Muscle weakness Prednisone, an oral glucocorticoid, increases the risk for gastric ulceration. Black, tarry stools can indicate gastrointestinal bleeding. This drug also increases the risk for infection. Clients should watch for and report early signs of infection, such as sore throat or fever. Prednisone also can result in fluid and electrolyte disturbances. Muscle weakness can indicate hypokalemia. Persistent cough is an adverse effect of angiotensin-converting enzyme inhibitors, such as Lisinopril (Zestril). Urinary retention can be an adverse effect of anticholinergic agents. (The Respiratory System Case Study)

A client is about to start taking aluminum hydroxide tablets to reduce gastric acid. Which instructions should you include when talking with the client about taking this drug? (Select all that apply.) A. Take the drug with a large meal B. Chew the tablets thoroughly C. Drink a glass of water after taking it D. Increase fluid and fiber intake E. Take it once daily

B, C, D B. Chew the tablets thoroughly C. Drink a glass of water after taking it D. Increase fluid and fiber intake Instruct clients to take aluminum hydroxide tablets one and three hours after meals, and at bedtime. The client should chew the tablets thoroughly (not swallow them whole) and follow the tablets or the liquid suspension with water to make sure it gets to the stomach promptly. This type of antacid can cause constipation, so make sure clients increase fluid and fiber intake and exercise more. Advise eating six small meals per day, instead of three large meals. Clients can take this drug up to six times a day. (The Gastrointestinal System Activity)

A client is about to start taking hydrochlorothiazide for hypertension. To assess for adverse effects this drug, you monitor for which during drug therapy? (Select all that apply.) A. Ototoxicity B. Hyponatremia C. Hypokalemia D. Dehydration E. Hypoglycemia

B, C, D B. Hyponatremia D. Hypokalemia D. Dehydration Common adverse effects of hydrochlorothiazide, a thiazide diuretic, including hyponatremia, hyperkalemia, and dehydration. Ototoxicity is more likely with loop diuretics', such as furosemide (Lasix) Hyperglycemia, not hypoglycemia, is a hydrochlorothiazide. (The Cardiovascular System Activity)

A client ask you about the difference between a generic drug and a trade- or brand-name drug which of the following are true regarding generic drugs? (Select all that apply.) A. Have the potential for abuse and dependence B. Have the same chemical composition as the brand-name drug C. May have several brand names D. May have several generic names E. Are usually less expensive than a brand-name drug

B, C, E B. Have the same chemical composition as the brand-name drug C. May have several brand names E. Are usually less expensive than a brand-name drug Generic and trade- or brand-name drugs have the same chemical composition and the same effects. Generic drugs are usually less expensive than brand-name drugs. Each drug has only one generic name. The trade name is the brand or proprietary name the manufacturer gives a drugs, so various manufacturers give the same drug different brand names. Brand names also differ with the various formulations or routes the same manufacturer provides. There is no difference in abuse potential between a generic and a brand-name drug. (Introduction into Pharmacology Activity)

A client is about to start taking liquid ferrous sulfate to treat iron deficiency anemia. Which of the following instructions should you include when talking with the client about taking this drug? (Select all that apply) A. Swish it in your mouth before swallowing B. Drink it through a straw C. Do not rinse your mouth after taking it D. Dilute it first with water E. Take it with food if needed.

B, D, E B. Drink it through a straw D. Dilute it first with water E. Take it with food if needed Because ferrous sulfate can stain the teeth, instruct the client to dilute the liquid form with water, drink it through a straw, and rinse her mouth after swallowing it. Food reduces the absorption of ferrous sulfate, so tell the client to take it on an empty stomach for best absorption. Clients who develop gastrointestinal distress from the drug can try taking it with food as long as they understand that this will reduce absorption and thus prolong therapy. (The Hematologic System Activity)

You are obtaining a health history from a client who is to start using a new ipratropium inhaler. Which of the following information from the clients history alerts you to take further action? (Select all that apply.) A. Seizure disorder B. Peanut allergy C. Depression treated with a monoamine oxidase inhibitor (MAOI) D. Hypothyroidism treated with synthetic thyroid hormone E. Glaucoma

B, E B. Peanut allergy E. Glaucoma Clients who are allergic to peanuts or do not take ipratropium an anticholinergic drug, because the medication preparation may contain peanut oil and soy lecithin. This drug requires caution with clients who have glaucoma because of the risk for increased intraocular pressure. Clients who have a seizure disorder or who take MAOIs or thyroid hormones can take ipratropium. The drug is inappropriate for clients who have prostatic hypertrophy. (The Respiratory System Activity)

A nurse is reinforcing teaching about benzodiazepine withdrawal with an older adult client who discontinued taking lorazepam after taking it for 3 months. Which of the following instructions should the nurse include? A. "Increase your fiber intake with each meal." B. "Have someone assist you with ADLs." C. "Watch for increased drowsiness." D. "Limit your daily fluid intake to 1,500 milliliters."

B. "Have someone assist you with ADLs.": The client who has lorazepam withdrawal can manifest tremors and dizziness, making ADLs difficult to perform. "Increase your fiber intake with each meal.": The client who is withdrawing from lorazepam does not need to increase dietary fiber because fiber is not associated with withdrawal manifestations. "Watch for increased drowsiness." The client who has lorazepam withdrawal can manifest insomnia. "Limit your daily fluid intake to 1,500 milliliters.": The client who has lorazepam withdrawal can manifest diaphoresis and should increase fluid intake to prevent dehydration.

A nurse is reinforcing teaching with a client who has a new prescription for timolol eye drops to treat glaucoma. Which of the following client statements indicates an understanding of the teaching? A. "I will rub my eyes for 10 seconds after putting in the medication." B. "I will look up when putting the medication into my eyes." C. "I will clean my eyes from the outer edge toward the nose before putting in the medication." D. "I will close my eyes tightly after putting in the medication."

B. "I will look up when putting the medication into my eyes.": The nurse should instruct the client to look up during administration of eye drops to help protect the cornea and minimize blinking. "I will rub my eyes for 10 seconds after putting in the medication.": The nurse should instruct he client to apply pressure to the lacrimal sac for 60 seconds after instilling the medication into his eyes to prevent systemic absorption of the medication, which can cause adverse effects such as hypotension. "I will clean my eyes from the outer edge toward the nose before putting in the medication.": The nurse should instruct the client to clean his eyes from the inner canthus toward the outer canthus to reduce the risk for introducing micro-organisms into the lacrimal duct. "I will close my eyes tightly after putting in the medication.": The nurse should instruct the client to close his eyes gently after instilling the medication. Closing his eyes tightly can result in loss of medication, leading to a reduction of desired therapeutic effects.

A nurse is caring for a client who has a prescription for a IM injection of penicillin G benzathine. The client asks why the injection must be given IM instead through the IV line. Which of the following responses should the nurse make? A. "The medication is more rapidly absorbed when given IM." B. "Your medication cant be given IV because it is not water-soluble." C. "You will experience less discomfort with an IM injection." D. "An IM injection allows more precise control of the medication level in your blood."

B. "Your medication cant be given IV because it is not water-soluble.": The nurse should inform the client this type of penicillin has poor water solubility and is never administered intravenous. "The medication is more rapidly absorbed when given IM.": The nurse should inform the client that this type of penicillin is absorbed slowly for several weeks, maintaining continuous low blood level. Medications given intravenous are absorbed faster than IM medications. "You will experience less discomfort with an IM injection.": The nurse should inform the client IM injections of this medication can cause discomfort at the injection site. "An IM injection allows more precise control of the medication level in your blood.": The nurse should inform the client that IV administration of a medication allows more precise control of the medication level in the blood.

A nurse is collecting data from a client who is taking exenatide to treat diabetes mellitus. For which of the following findings should the nurse withhold the exenatide dose and notify the provider immediately? A. Loss of appetite B. Abdominal pain C. Muscle weakness D. Heartburn

B. Abdominal pain: The greatest risk to this client is the development of pancreatitis as an adverse effect exenatide; therefore, the nurse should without the medication and immediately notify the provider of the clients abdominal pain. Manifestation of pancreatitis include severe, persistent abdominal pain with possible emesis. Loss of appetite: Loss of appetite indicates the client is at risk for impaired nutrition. However, another finding is the priority to report to the provider. Muscle weakness: Muscle weakness, or asthenia, indicates the client is at risk for fatigue and impaired self-care. However, another finding is the priority to report to the provider. Heartburn: Heartburn indicates that the client is at risk for gastroesophageal reflux. However, another finding is the priority to report to the provider.

A nurse is caring for a client who has genital herpes. Which of the following medication should the nurse expect to administer? A. Levofloxacin B. Acyclovir C. Ceftriaxone D. Metronidazole

B. Acyclovir: The client should receive acyclovir, an antiviral medication, to treat genital herpes by reducing symptoms and the rate of viral shedding. Levofloxacin: The nurse should expect to administer levofloxacin to treat chlamydia. Ceftriaxone: The nurse should expect to administer ceftriaxone to treat Lyme disease. Metronidazole: The nurse should expect to administer metronidazole to treat a clostridium difficile infection.

Which of the following information should you include when teaching a client about the use of albuterol inhaler for asthma management? A. Use it every 12 hours B. Use it periodically to prevent exercise-induced asthma C. Store the drug canister in the refrigerator D. Watch for an oral yeast infection

B. Use it periodically to prevent exercise-induced asthma Albuterol is a short-acting beta2 agonist that clients should use only as needed to stop an acute asthma exacerbation or 30 minutes before exercising to prevent exercise-induced bronchospasm. Salmeterol is an example of a drug that clients take every 12 hours for long-term asthma management. They should keep albuterol canisters away from heat and direct sunlight, but they should not refrigerate them. Inhaled glucocorticoids, such as beclomethasone, may cause oral yeast infections. They are not an adverse effect of albuterol. (The Respiratory System Activity)

A nurse is caring for a client who is having an acute asthma attack. Which of the following medications should the nurse administer first? A. Beclomethasone B. Albuterol C. Cromolyn D. Prednisone

B. Albuterol: According to evidence-based practice, the nurse should administer a short-acting beta2 agonist with a rapid onset when a client is experiencing an acute asthma attack. Therefore, the nurse should administer the albuterol prior to the other medication for prompt relief of the airway constriction. Beclomethasone: The nurse should administer beclomethasone as maintenance therapy for chronic asthma. However, evidence-based practice indicates that the nurse should administer another medication first. Cromolyn: The nurse should administer cromolyn as maintenance therapy for chronic asthma attacks. However, evidence-based practice indicates that the nurse should administer another medication first. Prednisone: The nurse should administer glucocorticoids, such as prednisone, to relieve airway inflammation. However, evidence-based practice indicates that the nurse should administer another medication first. The client client should take oral prednisone for 10 days following an acute attack to decrease inflammation.

A client is prescribed finasteride to treat benign prostatic hypertrophy. When providing instructions about the finasteride, the nurse should include which of the following information? A. Avoid high-fat foods B. Allow 6 to 12 months to see improvement C. Wear protective clothing and sunscreen D. Avoid activities that require alertness until drug effects are known

B. Allow 6 to 12 months to see improvement Finasteride may take 6 to 12 months to reduce symptoms. finasteride may be taken with or without food. It is not necessary to avoid fatty foods. Finasteride is unlikely to cause photosensitivity. Finasteride is unlikely to cause drowsiness or sedation. Advise client drug therapy is lifelong. (The Reproductive and Genitourinary Systems Activity)

Knowing the adverse drug reactions of ranitidine, the primary care providers assistant should instruct a client to watch for and report which of the following? A. Hyperactivity B. CNS Depression C. Hypertension D. Bone pain

B. CNS Depression Ranitidine, a histamine H2-receptor antagonist, can cause CNS depression, manifesting as drowsiness and confusion. The client should watch for and report any of these effects, as they might require adjustments in her drug therapy or possibly additional therapy. Ranitidine is unlikely to cause hyperactivity as there is more of a depressed CNS effect. It is unlikely to cause hypertension, but it can cause arrhythmias. The drug is also unlikely to cause bone pain, although it can cause headaches. (The Gastrointestinal System Case Study)

A nurse is collecting data from a client who has bacterial pneumonia and is taking ceftriaxone. Which of the following findings indicates a therapeutic effect of this medication? A. White blood cell count 10,500/mm^3 B. Clear, bilateral breath sounds C. Heart rate 110/min D. Tolerates small meal servings

B. Clear, bilateral breath sounds: The nurse should identify that wheezing and crackles are findings of bacterial pneumonia. A decreased in these manifestations indicates a therapeutic effect of the medication. White blood cell count 10,500/mm^3: The nurse should identify that a WBC count of 10,500/mm^3 is above the expected reference range, which indicates a continuation of the infection. Heart rate 110/min: The nurse should identify that tachycardia is a manifestation of pneumonia. This heart rate is above the expected reference range and, therefore, is not an indication that the medication has been effective. Tolerates small meal servings: The nurse should identify that pneumonia decreases a clients energy and can interfere with maintaining adequate nutrition. The ability to tolerate small meal servings is not indication that the medication has been effective.

A client comes to an urgent care clinic and announces with great enthusiasm, "I am an expert at all things medical as they apply to me, and I require zolpidem." The clients pupils are dilated, along with elevated heart rate and blood pressure level. The nurse should suspect intoxication with which of the following substances? A. Alcohol B. Cocaine C. Barbiturates D. Heroin

B. Cocaine: The client who has cocaine intoxication typically has tachycardia, elevated blood pressure, dilated pupils, and displays delusions. This clients behavior and physiological data indicated cocaine intoxication. Alcohol: The client who has alcohol intoxication typically has slurred speech, drowsiness, impaired judgment, irritability, and decreased blood pressure. Barbiturates: The client who ahs barbiturate toxicity typically has respiratory depression, contracted pupils, drowsiness, impair judgment, irritability, and decreased blood pressure. Heroin: The client who has heroin toxicity typically has slurred speech, drowsiness, constricted pupils, and decreased blood pressure.

A nurse is collecting data from a client who is taking oral amoxicillin to treat respiratory infection. The nurse should monitor the client for which of the following manifestations as a adverse effect of the medication? A. Hearing loss B. Diarrhea C. Bruising D. Tendonitis

B. Diarrhea: The nurse should monitor the client for diarrhea. A clostridium difficile infection is an adverse effect of antibiotics, including amoxicillin. Hearing loss: The nurse should monitor for hearing loss in clients who are taking aminoglycosides, such as amikacin. Bruising: The nurse should monitor for bruising and bleeding in clients who are taking cephalosporins, such as ceftriaxone. Tendonitis: The nurse should monitor for tendonitis on tendon rupture in clients who are taking fluoroquinolones, such as ciprofloxacin.

You are about to administer ondansetron to a client to prevent anesthesia-induced nausea and vomiting. You prepare to monitor the client for which adverse effects of this drug? A. Bronochospasm B. Dizziness C. Hypertension D. Anxiety

B. Dizziness ondansetron, a serotonin antagonist, can cause dizziness, lightheadedness, and sedation. It is unlikely to cause bronchospasm, hypertension, or anxiety. However, it can cause a headache, so recommend an over-the-counter analgesic for headache pain. This drug can also cause ECG changes in clients and should be used cautiously with clients who have abnormal heart rhythm or electrolyte imbalance. (The Gastrointestinal System Activity)

You are talking with a client who is taking losartan for hypertension. She asks you about stopping contraception and starting a family. You inform the client that losartan is unsafe to take during pregnancy due to which risk? A. Macrosomia B. Fetal injury C. Gestational diabetes D. Thromboembolic disease

B. Fetal injury Angiotensin II receptor blockers such as losartan are teratogenic drugs that may cause fetal harm, such as slowed development and renal failure. Women should not take them during the second and third trimesters of pregnancy. The drug does not appear to cause fetal macrosomia, gestational diabetes, or maternal blood clots. (The Cardiovascular System Activity)

A nurse is reviewing medication prescriptions for a group of clients. The nurse should recognize that which of the following prescriptions can result in a medication administration error? A. Penicillin G benzathine 1.2 million units IM daily B. Furosemide 10.0 mg PO daily C. Albuterol 2.5 mg 2 inhalations every 6 hr as needed for shortness of breath D. Insulin glargine 15 units subcutaneous daily at bedtime

B. Furosemide 10.0 mg PO daily: The nurse should avoid using a trialing zero following a whole number. This prescription can result in a medication error because the nurse can mistake the dosage as 100 mg instead of 10 mg because the decimal point is not always recognized. Penicillin G benzathine 1.2 million units IM daily: The nurse should recognize that this is written correctly and should not result in a medication administration error. Albuterol 2.5 mg 2 inhalations every 6 hr as needed for shortness of breath: The nurse should recognize that this is written correctly and should not result in a medication administration error. Insulin glargine 15 units subcutaneous daily at bedtime: The nurse should recognize that this is written correctly and should not result in a medication administration error.

A nurse is reviewing the medical administration record of a client who has a history of Stevens-Johnson syndrome when taking sulfamethoxazole-trimethoprim. Which of the following medications should the nurse identify as contraindicated for this client? A. Prednisone B. Furosemide C. Lansoprazole D. Digoxin

B. Furosemide: A client who has a history of Stevens-Johnson syndrome when taking sulfonamides is at risk for an allergic reaction to furosemide because the two medications are chemically related. The client should also avoid thiazide diuretics and sulfonylurea-type oral hypoglycemic agents. Prednisone: A client who has a history of Stevens-Johnson syndrome when taking sulfonamides is not at risk for an allergic reaction with prednisone. Lansoprazole: A client who has a history of Stevens-Johnson syndrome when taking sulfonamides is not at risk for allergic reaction with lansoprazole. Digoxin: A client who has history Stevens-Johnson syndrome when taking sulfonamides is not at risk for an allergic reaction with digoxin.

A nurse is collecting data from a parent of an 11-year-old child who has previously received all the immunizations in accordance with the CDC Recommended Immunization Schedule. Which of the following immunization should the nurse plan to administer to this child at this age? A. Hepatitis A (HepA) B. Human papilloma virus (HPV) C. Varicella (VAR) D. Inactivated poliovirus (IPV)

B. Human papilloma virus (HPV): This is voluntary immunization that the CDC's Advisory Committee on Immunization Practices recommends for children between the ages of 11 and 12 years of age. The immunization is intended to prevent the development of human papilloma virus. Certain strains of this virus can lead to genital warts and cancers. Hepatitis A (HepA): Having received all previous immunizations according to the CDC Schedule, the child should have received the HepA immunization series between the ages of 12 and 24 months of age. Varicella (VAR): Having received all previous immunizations according to the CDC Schedule, the child should have received the varicella A immunization series between the ages of 12 months and 6 years of age. Inactivated poliovirus (IPV): Having received all previous immunizations according to the CDC Schedule, the child should have received the IPV immunizations series between the ages of 2 months and 6 years of age.

A nurse is caring for a client who has a new prescription for eplerenone to treat hypertension. The nurse should monitor for which of the following adverse effects of this medication? A. Hematuria B. Hypernatremia C. Hyperkalemia D. Constipation

B. Hyperkalemia: The nurse should identify that eplerenone can place the client at risk for increased potassium levels because eplerenone can cause potassium retention. Hematuria: The nurse should identify that eplerenone can cause vaginal bleeding. not blood in the urine. Hypernatremia: The nurse should identify that eplerenone can cause decreased sodium levels, not increased sodium levels. Constipation: The nurse should identify that eplerenone can cause diarrhea, not constipation.

A nurse is assisting with the care of a client who is in preterm labor and is receiving magnesium sulfate by continuous IV infusion. Which of the following findings indicates magnesium toxicity? A. Urinary output 60 mL/hr B. Hyporeflexia C. Respirations 14/min D. Tachycardia

B. Hyporeflexia: Magnesium sulfate and depresses neuromuscular activity, causing muscle weakness and paralysis. Therefore, the nurse should identify hyporeflexia as an indication of magnesium toxicity and report it to the charge nurse. Urinary output 60 mL/hr: A urine output of less than 25 to 30 mL/hr can indicate magnesium toxicity. Respirations 14/min: Respirations less than 12/min can indicate magnesium toxicity. Prior to initial administration of magnesium sulfate, the nurse should ensure a client has a respiratory rate of 16/min or greater. Tachycardia: Bradycardia is an adverse effect and can indicate magnesium toxicity.

A nurse is caring for a client who has kidney failure and has been taking epoetin alfa. Which of the following is a therapeutic effect of this medication? A. Decreased BUN B. Increased Hgb C. Decreased leukocyte production D. Increased platelet production

B. Increased Hgb: Epoetin alfa is used to elevate erythrocyte count for clients who have kidney failure. An increased Hgb is the desired therapeutic effect of this medication. Decreased BUN: Epoetin alfa does not correct renal deterioration. A decrease in BUN is not expected effect of this medication. Decreased leukocyte production: Epoetin alfa does not have an effect on leukocyte production. Increased platelet production: Epoetin alfa does not have an effect on platelet production.

A nurse is caring for a client who has a history of psychosis and is taking chlorpromazine. Which of the following actions should the nurse take to counteract the adverse effects of this medication? A. Suggest that the client apply antiperspirant more frequently. B. Inform the client to apply sunblock before going outside. C. Give the client a list of over-the-counter antidiarrheal medications. D. Recommend that the client take the medication on an empty stomach.

B. Inform the client to apply sunblock before going outside.: The nurse should inform the client to apply sunblock, which will counteract the adverse effects of photosensitivity. Chlorpromazine increases skins sensitivity to ultraviolet light causing temporary pigmentation changes that increases the risk of sunburn. Suggest that the client apply antiperspirant more frequently.: Chlorpromazine has the adverse effect of reduced perspiration. Tricyclic antidepressants, rather than antipsychotics, can cause excessive sweating. Give the client a list of over-the-counter antidiarrheal medications.: The anticholinergic properties of antipsychotic drugs can cause constipation, not diarrhea. Recommend that the client take the medication on an empty stomach.: The client should take chlorpromazine with food or fluid to minimize gastrointestinal distress.

A nurse in a providers office is reviewing the immunization of records of a 12-month-old infant who is immunocompromised. Which of the following vaccines should the nurse identify as contraindicated for this client? A. Hepatitis A (HepA) B. Measles, mumps, rubella (MMR) C. Pneumococcal conjugate (PCV13) D. Haemophilus B conjugate (Hib)

B. Measles, mumps, rubella (MMR): Although most infant should receive the MMR vaccine between the ages of 12 months and 15 months, the nurse should identify that the MMR vaccine is composed of live viruses and is contraindicated for a infant who is immunocompromised. Hepatitis A (HepA): The nurse should identify that the HepA vaccine series should be administered between the ages of 12 months and 23 months. It does not contain a live virus and is not contraindicated for an infant who is immunocompromised. Pneumococcal conjugate (PCV13): The nurse should identify that the PCV13 vaccine series should be administered between the ages of 2 months and 15 months. It does not contain a live virus and is not contraindicated for an infant who is immunocompromised. Haemophilus B conjugate (Hib): The nurse should identify that the Hib vaccine series should be administered between the ages of 2 months and 15 months. It does not contain a live virus and is not contraindicated for an infant who is immunocompromised.

A nurse is caring for a client who has a prescription for hydromorphone 2 mg immediate release tablet by mouth. The nurse administers a 4 mg immediate release tablet by mistake. Which of the following actions should the nurse take first? A. Document the facts about the error in the medication administration record. B. Obtain the clients vital signs and level of consciousness. C. Prepare an incident report. D. Report the error to the provider and charge nurse.

B. Obtain the clients vital signs and level of consciousness.: The first action the nurse should take when using the nursing process is to collect data from the client, such as vital signs and level of consciousness. After a medication error, the first thing the nurse should do is evaluate the client for adverse effects. A double dose of hydromorphone increases the clients risk for oversedation and respirator depression. Document the facts about the error in the medication administration record.: The nurse should document only factual information about the error in the medication administration record so that other members of the health care team can continue to monitor the client for adverse effects. However, there is another action the nurse should take first. Prepare an incident report.: The nurse should prepare an incident report to the document the medication error, the clients condition, and actions taken by the nurse. However, there is another action the nurse should take first. Report the error to the provider and charge nurse.: The nurse should inform the provider and the charge nurse of the medication error to ensure the continued safety of the client. The provider might prescribe a reversal agent for the excess dose of opioids. However, there is another action the nurse should take first.

A nurse is collecting data from a client who is postoperative and taking morphine for pain. Which of the following finds is the priority for the nurse to report to the provider? A. Constipation B. Oxygen saturation 87% C. Vomiting D. Urinary output 25 mL over 1 hr

B. Oxygen saturation 87%: When using the airway, breathing, circulation approach to the client care, the nurse determines that the priority finding is an oxygen saturation of 87%, which is a manifestation of respiratory depression and should be reported to the provider. Constipation: The nurse should monitor and treat the client for constipation; however, there is another finding that is the priority for the nurse to report. Vomiting: the nurse should monitor and treat the client for vomiting; however, there is another finding that is the priority for the nurse to report. Urinary output 25 mL over 1 hr: The nurse should monitor and treat the client for urinary retention; however, there is another finding that is the priority for the nurse to report.

A client is about to start taking warfarin to prevent venous thrombosis. You should explain to the client that taking this drug initially requires daily blood samples to monitor the results of which of the following laboratory tests? A. Fibrinogen B. PT/INR C. Platelets D. aPTT

B. PT/INR At the start of warfarin therapy, the prescriber should monitor prothrombin time (PT) and international normalized ratio (INR) daily and adjust the dosage to maintain an INR of 2 to 3. Monitor the client for indication of bleeding such as abdominal pain, black tarry stools, or nosebleeds. Warfarin does not affect fibrinogen level, a parameter to assess prior to alteplase therapy. Aspirin is an example of a drug that inhibits platelet aggregation. Warfarin does not affect fibrinogen level, a parameter to assess prior to alteplase therapy. Warfarin does not affect activated partial thromboplastin time (aPTT), a parameter to assess every 4 to 6 hr initially and then daily for clients receiving continuous IV heparin. (The Hematologic System Activity)

A client is prescribed clomiphene for infertility. The health care provider instructs the client on clomiphene to monitor for which of the following? A. Shortness of breath B. Pelvic pain C. Diarrhea D. Fever

B. Pelvic pain Lower abdominal or pelvic pain may indicate ovarian hyperstimulation and may lead to ruptured ovarian cyst. Clients should be monitored carefully for ovarian enlargement and seek medical attention immediately for sudden or severe pelvic pain. Shortness of breath is unlikely caused by clomiphene. Pulmonary emboli from thrombosis may be caused by combination oral contraceptives and estrogens. Clomiphene is unlikely to cause a fever. It may cause visual disturbances and dizziness. (The Reproductive and Genitourinary Systems Activity)

A nurse is collecting data from a female client who has been taking propylthiouracil (PTU) for 2 months to treat Graves disease. Which of the following findings should the nurse recognize as an indication that the medication is effective? A. Weight loss B. Pulse 82/min C. Respiratory rate 22/min D. Decreased menstrual flow

B. Pulse 82/min: Tachycardia is a manifestation of hyperthyroidism. The nurse should identify that a pulse of 82/min is within the expected reference rang of 60 to 100/min, indicating that the medication is effective. Weight loss: The nurse should identify that weight loss is a manifestation of hyperthyroidism, which indicated the medication is not effective. Respiratory rate 22/min: The nurse should identify that tachypnea is a manifestation of hyperthyroidism, which indicates the medication is not effective. Decreased menstrual flow: The nurse should identify that a amenorrhea is a manifestation of hyperthyroidism, which indicates the medication is not effective.

A nurse is reinforcing teaching with a client who has hypertension and a new prescription for spironolactone. Which of the following instructions should the nurse include in the teaching? A. Increased foods high in zinc. B. Restrict foods high in potassium. C. Restrict foods high in vitamin K. D. Increase foods high in magnesium.

B. Restrict foods high in potassium: The nurse should instruct the client that spironolactone is a potassium-sparing diuretic, which can cause hyperkalemia. Therefore, the client should restrict foods that are high in potassium and salt substitutes that contain potassium. Increase foods high in zinc: The nurse should instruct the client that spironolactone does not affect the clients zinc level. Restrict foods high in vitamin K: The nurse should instruct the client that spironolactone does not affect the clients vitamin K level. Increase foods high in magnesium: The nurse should instruct the client that spironolactone does not affect the clients magnesium level.

A nurse is reinforcing teaching about comfort measures with the parent of a 10-year-old child who has a viral infection. The nurse should plan to tell the parent that aspirin is contraindicated because of the risk for which of the following conditions? A. Juvenile idiopathic arthritis B. Reye's syndrome C. Glomerulonephritis D. Iron-deficiency anemia

B. Reye's syndrome: Aspirin is contraindicated for children and adolescents who have a viral illness because it is associated with the development of Reye's syndrome. There is a risk for children and adolescents to develop Reye's syndrome if they take a aspirin following a viral illness. Juvenile idiopathic arthritis: There is no association between taking aspirin for viral illness in childhood and developing juvenile idiopathic arthritis. Glomerulonephritis: There is no association between taking aspirin for a viral illness in childhood and developing glomerulonephritis. Iron-deficiency anemia: There is no association between taking aspirin for a viral illness in childhood and developing iron-deficiency anemia.

A nurse is reinforcing teaching with a client who has a new prescription for omeprazole oral capsule. Which of the following instructions should the nurse include? A. Take the medication at bedtime. B. Swallow the medication whole. C. Take the medication with food. D. Avoid antacids when taking this medication.

B. Swallow the medication whole.: The nurse should instruct the client to swallow the capsules or tablets whole and not chew or crush them. Omeprazole, a proton pump inhibitor, blocks the secretions of gastric avid. It is available in delayed-release capsules and over the counter in delayed-release tablets, as well as suspensions and powders. Take the medication at bedtime.: The client should take omeprazole in the morning, prior to the first meal of the day, to increase the absorption of the medication. Take the medication with food.: Food can reduce the absorption of omeprazole, a proton pump inhibitor. Therefore, the client should avoid taking the medication with food. Avoid antacids when taking this medication.: Antacids do not alter the effectiveness of omeprazole. Therefore, the client does not need to avoid antacids when taking this medication.

Recognizing the adverse effects of terbutaline, the nurse should monitor the client for which of the following? A. Hypoglycemia B. Tachycardia C. Hyperkalemia D. Wheezing

B. Tachycardia Terbutaline, a beta2-adrenergic agonist, may cause tachycardia in both the client and in the fetus. Monitor vital signs carefully, including the fetal monitor for tachycardia. Hyperglycemia, rather than hypoglycemia, is an adverse effect of terbutaline. Hypokalemia, rather than hyperkalemia, may occur when taking terbutaline. Terbutaline relaxes bronchial constriction and is used to treat asthma; therefore, wheezing is not an adverse effect of this drug. (The Reproductive and Genitourinary Systems Case Study)

A client is about to start taking sustained release glipizide to treat type 2 diabetes mellitus. Which of the following instructions should you include when talking with the client about taking this drug? A. Chew the tablet completely before swallowing it B. Take it once a day, 30 min before breakfast C. Take it in the evening before bedtime D. Drink 16 oz of water right after taking it

B. Take it once a day, 30 min before breakfast Instruct the client to take sustained-release glipizide, a sulfonylurea, 30 min before the first meal of the day. Tell the client not to chew or crush the sustained-release tablets. It is not necessary to drink 16 oz of water after taking the drug, but it is not harmful to do so, either. Because glipizide peaks in 1 to 2 hr, it is inappropriate to take it at bedtime and appropriate to have a meal after taking it. Be sure to monitor the client for signs of hypoglycemia, such as diaphoresis and tachycardia, sweating, hunger, weakness, dizziness, tremor, tachycardia, anxiety. Also, monitor heart rate and ECG. Instruct the client to watch for and report palpitations. (The Endocrine System Activity)

For rapid relief of urinary obstruction due to prostatic enlargement, the health care provider should choose which of the following drugs? A. Sildenafil B. Tamsulosin C. Finasteride D. Oxybutynin

B. Tamsulosin Tamsulosin, an alpha1-adrenergic antagonist,, provides smooth muscle relaxation and rapid relief of urinary obstruction. It is often used in combination with finasteride. Finasteride may take 6 to 12 months for effects. Sildenafil, a PDE-5 inhibitor, is used to treat erectile dysfunction. Oxybutynin, an anticholinergic, is used to treat urinary incontinence and neurogenic bladder. (The Reproductive and Genitourinary Systems Activity)

A nurse is reinforcing teaching about immunizations with a client who is pregnant. Which of the following vaccines should the nurse include in the teaching as safe to administer during pregnancy? A. Herpes zoster B. Tetanus and diphtheria (Td) C. Varicella D. Measles, mumps, and rubella (MMR)

B. Tetanus and diphtheria (Td): The nurse should include that either the Td vaccine or the tetanus, diphtheria, and pertussis (Tdap) vaccine is safe for administration during pregnancy. Tdap is the preferred vaccine. Herpes zoster: The herpes zoster vaccine is contraindicated during pregnancy. Adults 60 years or older should receive the herpes zoster vaccine. Varicella: The varicella vaccine is contraindicated during pregnancy because it contains live virus. Measles, mumps, and rubella (MMR): The MMR vaccine is contraindicated during pregnancy because it contains a combination of three live viruses.

While instructing a client about using isosorbide to treat angina, you advise the client to avoid which of the following? A. Milk B. Caffeine C. Alcohol D. Grapefruit juice

C. Alcohol Alcohol may increase the hypotensive effects of isosorbide, an antianginal agent, and thus increase the risk of cardiovascular collapse. Milk, caffeine, and grapefruit juice are not contraindicated for clients taking this drug. Milk, however can interfere with the action of some forms of supplemental iron. Caffeine can worsen some of the adverse effects of beta2-adrenergic agonists, and grapefruit juice can interfere with the metabolism of many drugs, including calcium channel blockers. (The Cardiovascular System Activity)

Next, the provider tells the client that she will also take albuterol via a metered-dose inhaler (MDI). When instructing the client about using the MDI, you should include which of the following information? A. Take a short, quick breathe before using the inhaler B. Wait at least 1 minute before inhaling with second puff C. Hold down the canister of the inhaler while inhaling rapidly D. block one nasal passage while inhaling

B. Wait at least 1 minute before inhaling with second puff Most clients are prescribed two puffs of the inhaler with each dose. Waiting at least 1 minute, or the prescribed interval, before taking the second puff allows time for the bronchodilation, which may increase the absorption of the next puff. The client should take a slow, deep breath lasting for 3 to 5 seconds to facilitate delivery to the air passages. It is not necessary to block either nasal passage while inhaling; in fact, it could interfere with optimal inhalation of the drug. (The Respiratory System Case Study)

A nurse is reinforcing teaching with the parent of a preschooler who has otitis media. The child has a low-grade fever and irritability for 2 days. Which of the following instructions should the nurse include in the teaching? A. "Administer amoxicillin twice a day for 3 days." B. "Apply cold packs every 4 hours for relief of pain." C. "Give acetaminophen as needed for discomfort and fever." D. "Return to the office in 72 hours for a follow-up appointment."

C. "Give acetaminophen as needed for discomfort and fever.": The nurse should instruct the parent to administer analgesics, such as acetaminophen or ibuprofen, to decrease discomfort and fever related to otitis media. "Administer amoxicillin twice a day for 3 days.": The nurse should instruct the parent to administer the full course of antibiotics, which is expected to be 5 to 7 days for mild to moderate infection and 10 days for severe infection. "Apply cold packs every 4 hours for relief of pain.": The nurse should instruct the parent to apply heat to the ear while the child lies on affected side to reduce discomfort. This position promotes drainage to exudate. "Return to the office in 72 hours for a follow-up appointment.": The nurse should instruct the parent to return for a follow-up appointment when the antibiotic therapy has been completed. This allows for determining the effectiveness of the treatment and identification of complications.

A nurse is reinforcing teaching with a client who has HIV and a new prescription for zidovudine. Which of the following client statements should indicate to the nurse an understanding of the teaching? A. "I can have unprotected sex after 6 months of taking this medication." B. "I can expect to have constipation while taking this medication." C. "I will be sure to have my blood tested for anemia." D. "My fingers might feel numb after I start therapy."

C. "I will be sure to have my blood tested for anemia.": Zidovudine can cause severe anemia and neutropenia. The client should have blood tests performed before treatment begins and have continued monitoring during the course of treatment. "I can have unprotected sex after 6 months of taking this medication.": The client should continue to use protection during sexual activity even if the plasma HIV RNA is undetectable after taking zidovudine. "I can expect to have constipation while taking this medication.": Zidovudine can cause GI disturbances, such as diarrhea, abdominal pain, nausea, and vomiting. "My fingers might feel numb after I start therapy.": Numbness and tingling are not adverse effects of zidovudine. The clients nail beds might have changes to pigmentation while taking this medication.

A nurse is reinforcing teaching with a client about the use of sublingual nitroglycerin for chest pain. Which of the following statements by the client indicates an understanding of the teaching? A. "I will store my nitroglycerin tablets in my care glove box when traveling." B. "I will take a nitroglycerin tablet 60 minutes prior to exercise." C. "I will call 911 if my chest pain is not relieved within 5 minutes of taking nitroglycerin." D. "If I get a headache after taking nitroglycerin tablet, I should report to it to my doctor immediately."

C. "I will call 911 if my chest pain is not relieved within 5 minutes of taking nitroglycerin.": The nurse should instruct the client to call 911 if chest pain is not relieved within 5 min after taking nitroglycerin. Chest pain not relieved by nitroglycerin can be an indication that the client is having a myocardial infarction. The client should continue to take two more nitroglycerin tablets every 5 min for continued chest pain, while waiting for emergency response to arrive. "I will store my nitroglycerin tablets in my care glove box when traveling.": The nurse should instruct the client to store nitroglycerin at room temperature, not in the extreme heat or cold inside a cars glove box. "I will take a nitroglycerin tablet 60 minutes prior to exercise.": The nurse should instruct the client to take a nitroglycerin tablet prophylactically 5 to 10 min prior to exercise because sublingual nitroglycerin has an onset of 1 to 3 min with a duration of up to 60 min. "If I get a headache after taking nitroglycerin tablet, I should report to it to my doctor immediately.": The nurse should instruct the client that headaches are common adverse effect of nitroglycerin and to use a mild analgesic to relieve the headache.

A nurse is reinforcing teaching with a client who has prescription for propranolol. Which of the following information should the nurse include in the teaching? A. "If you miss a dose, double the next scheduled dose." B. "Discontinue this medication if lightheadedness occurs." C. "If your pulse rate is less than 50 beats per minute, notify your provider." D. "This medication can cause heat intolerance."

C. "If your pulse rate is less than 50 beats per minute, notify your provider.": The nurse should instruct the client to check his pulse before taking the medication and to withhold the medication if his pulse is than 50/min. the client should notify his provider bradycardia is a common adverse effect of beta blockers. "If you miss a dose, double the next scheduled dose.": The nurse should instruct the client to take a missed soon as soon as he notices and prior to 4 hr before the next dose is scheduled. "Discontinue this medication if lightheadedness occurs.": The nurse should instruct the client that lightheadedness and dizziness are adverse effects of this medication, and the client should avoid driving or other hazardous activities until the effects of the medication are known. the nurse should instruct the client to avoid abrupt discontinuation of the medication as this can cause life-threatening arrhythmias. "This medication can cause heat intolerance.": The nurse should instruct the client that beta blockers, such as propranolol, can cause cold intolerance.

A nurse is reinforcing teaching with a client who has a new prescription for vitamin B12 intranasal to treat malabsorption syndrome. Which of the the following instructions should the nurse include in the teaching? A. "Take this medication once per day." B. "Drink a hot liquid after administering the medication." C. "Massage your nose gently after medication administration." D. "Store this medication in the refrigerator."

C. "Massage your nose gently after medication administration.": The nurse should instruct the client to massage her nose gently to increase absorption of the medication. "Take this medication once per day.": The nurse should instruct the client to administer vitamin B12 intranasally once per week. Oral vitamin B12 formulations require daily dosing. "Drink a hot liquid after administering the medication.": The nurse should instruct the client to avoid hot liquids within 1 hr before and after taking B12. Hot liquids or spicy foods can cause rhinitis, which limits the effectiveness of the medication. "Store this medication in the refrigerator.": The nurse should instruct the client to store vitamin B12 at room temperature.

A nurse is reinforcing discharge teaching with a client who has a prescription for a metered-dose inhaler (MDI). Which of the following information should the nurse include in the teaching? A. "Wait for 15 seconds between each puff of the same medication." B. "Hold your breath for 5 seconds after inhaling the medication." C. "Take a slow, deep breath lasting 3 to 5 seconds after releasing the medication." D. "Roll the canister between your hands for 10 to 15 seconds to mix the medication."

C. "Take a slow, deep breath lasting 3 to 5 seconds after releasing the medication.": The client should take a slow, deep breath lasting 3 to 5 seconds to allow the medication to be distributed deeply into her lungs. "Wait for 15 seconds between each puff of the same medication.": The client should wait 20 to 60 seconds between puffs of the same medication. If two different medications are administered, the client should wait 2 to 5 min between administrations. "Hold your breathe for 5 seconds after inhaling the medication.": The client should hold her breath for at least 10 seconds after inhaling the medication to be distributed deeply into her lungs. "Roll the canister between your hands for 10 to 15 seconds to mix the medication." The client should shake the canister vigorously for 3 to 5 seconds to ensure that fine particles are aerosolized.

An older adult client who has terminal cancer reports pain at a level of 5 using a 1 to 10 pain scale. The client has a prescription for morphine 15 mg orally every 4 hr. The clients children express concern that the client is receiving to much of the medication. Which of the following responses should the nurse make? A. "Clients who have receive this medication orally have a lower risk for addiction." B. "Additional doses will not needed because this medication is given on a fixed schedule." C. "The dose should remain constant to prevent breakthrough pain." D. "We can switch from oral administration to rectal administration."

C. "The dose should remain constant to prevent breakthrough pain.": Fixed or scheduled dosing around the clock offers the best pain control for clients who have severe and persistent pain. "Clients who have receive this medication orally have a lower risk for addiction.": The risk for addiction does not change with the route of administration. "Additional doses will not needed because this medication is given on a fixed schedule.": If a client who has terminal cancer is receiving morphine on a fixed schedule and experiences breakthrough pain, she may receive additional doses of the medication. "We can switch from oral administration to rectal administration.": Rectal administration is used for clients who cannot take the drug orally. However, changing the route of administration will not affect the amount of medication provided.

A nurse is reinforcing teaching about nicotine polacrilex gum with a client who smokes three packs of cigarettes per day. Which of the following statements should the nurse include in the teaching? A. "You can drink caffeinated beverages while chewing the gum." B. "You can chew two pieces of gum when you have the urge to smoke." C. "You should chew the gum for a full 30 minutes before discarding." D. "You should wait 5 minutes after eating to chew the gum."

C. "You should chew the gum for a full 30 minutes before discarding.": The nurse should instruct the client to chew the gum slowly for a full 30 min before discarding. The full dose of nicotine from the gum occurs within 15 to 30 min. "You can chew two pieces of gum when you have the urge to smoke.": The nurse should instruct the client to chew one piece of gum when having the urge to smoke to avoid adverse effects including nausea, throat discomfort, and hiccups. "You should wait 5 minutes after eating to chew the gum.": The nurse should instruct the client to wait 15 min after eating to chew the gum. Food and beverages can reduce nicotine absorption. "You should wait 5 minutes after eating to chew the gum.": The nurse should instruct the client to wait 15 min after eating to chew the gum. Food and beverages can reduce nicotine absorption.

You note that a primary care provider prescribed morphine sulfate, an opioid agonist, to relieve a clients postoperative pain. which of the following actions describes the action of an agonist on receptor? A. Destroys the receptor B. Competes with the receptor C. Activates the receptor D. Blocks the receptor

C. Activates the receptor Agonists bind with receptor sites on a cell and activate them to produce a response. In contrast, antagonist block drugs from receptor sites, blocking the drugs intended reaction. Antagonists that compete with an agonist for cell receptor sites are called competitive antagonists. Drugs typically do not destroy the receptor sites on cells. (Introduction into Pharmacology Activity)

The provider talks with the client about clomiphene therapy. Which of the following instruction should the provider include? A. Take it once a day until they can confirm pregnancy B. Begin takin git on the first day and stop on the last day of menstruation C. Begin 5 days after menstruation beings and take it every day for the next 5 days. D. Take it once, 17 to 19 days after the onset of menstruation.

C. Begin 5 days after menstruation beings and take it every day for the next 5 days. The client should start taking clomiphene 5 days after her menstruation begins and continue taking it everyday for the next 5 days. She may repeat this dosing schedule at 30-day intervals for three cycles. The primary care provider might recommend taking human chorionic gonadotropin 7 to 9 days after clomiphene use or 17 to 19 days after menstruation begins. (The Reproductive and Genitourinary Systems Case Study)

A nurse is reviewing the medical record of a client who has a new prescription for dimenhydrinate to treat motion sickness. Which of the following disorders in the clients medical record should the nurse report to the provider? A. Major depressive disorder B. Diabetes mellitus C. Benign prostatic hyperplasia D. Meniere's disease

C. Benign prostatic hyperplasia: Clients who have benign prostatic hyperplasia might have urinary hesitancy and retention and, therefore, should not take dimenhydrinate due to the anticholinergic adverse effects of the medication. Major depressive disorder: Clients who have major depressive disorder can take dimenhydrinate, an antihistamine. Diabetes mellitus: Clients who have diabetes mellitus can take dimenhydrinate, an antihistamine. Meniere's disease: Clients who have Meniere's disease can take dimenhydrinate, an antihistamine, as a form of treatment.

A nurse is reinforcing teaching with a client who experiences migraine headaches and has a new prescription for sumatriptan. The nurse should instruct the client to report which of the following manifestations to the provider as an adverse effect of this medication? A. Insomnia B. Photophobia C. Chest tightness D. Respiratory depression

C. Chest tightness: The nurse should instruct the client to report chest pain or tightness to the provider because this can be a manifestation of a vasospastic response. Insomnia: The nurse should instruct the client that drowsiness is an adverse effect of sumatriptan and to use caution when driving or operating machinery. Photophobia: The nurse should instruct the client that visual changes are an adverse effect of sumatriptan. However, photophobia, or sensitivity to light, can be a manifestation of the migraine itself. Respiratory depression: The nurse should instruct the client that bronchospasm is an adverse effect of sumatriptan, which can result in wheezing. However, respiratory depression is not associated with this medication.

A client is about to start taking omeprazole to treat a duodenal ulcer. Which of the following instructions should you include when talking with the client about taking this drug? A. Take with food B. Avoid using aluminum-based antacids C. Consume adequate amounts of fluids D. Do not drink grapefruit juice

C. Consume adequate amounts of fluids Food can reduce the absorption of omeprazole, so instruct to take this drug 1 hr before a meal and with plenty of fluids. Antacids are unlikely to interfere with the absorption of omeprazole and might help reduce gastric distress. Grapefruit juice interacts with many different drugs, including cylosporine (Sandimmune), midazolam (Versed), and lovastatin (Mevacor), but it is unlikely to affect the metabolism of ompeprazole. (The Gastrointestinal System Activity)

Recognizing the potential for a drug interaction between the diuretic, furosemide, and the cardiac glycoside, digoxin, the primary care provider plans to monitor this client for which adverse effect? A. Hyperkalemia B. Ototoxicity C. Digoxin toxicity D. Hypoglycemia

C. Digoxin toxicity Furosemide, a high-ceiling loop diuretic, can cause hypokalemia, not hyperkalemia. Hypokalemia, in turn increases the risk for digoxin toxicity, which has the potential for causing serious ventricular dysrhythmias. Clients taking these two drugs should also take potassium supplements to reduce their risk for digoxin toxicity. Furosemide can cause ototoxicity, with the risk of increasing with the use of other ototoxic drugs, such as gentamicin (Garamycin), an aminoglycoside. But ototoxicity is not a result of an interaction between these two drugs. Furosemide can cause hyperglycemia, which makes monitoring of blood glucose essential for clients taking this drug. (The Cardiovascular System Case Study)

A primary care provider then prescribes quinidine gluconate as an oral antidysrhythmic agent. A health care professional should question the use of quinidine for client taking digoxin, due to a possible drug interaction that can result in which adverse effect? A. Liver toxicity B. Quinidine toxicity C. Digoxin toxicity D. Cardiotoxicity

C. Digoxin toxicity Quinidine, a class 1A antidysrhythmic drug, may double digoxin levels by significantly reducing the elimination of digoxin. This puts clients at high risk for digoxin toxicity. Quinidine can cause liver dysfunction, but this adverse effect is not a result of its interaction with digoxin. High levels of quinidine can cause quinidine toxicity and severe cardiotoxicity, but these adverse effects are not a result of its interaction with digoxin. (The Cardiovascular System Case Study)

A nurse is reinforcing teaching with a client following placement of a cast for a fractured ankle. The client is to take oxycodone for pain management. The nurse should instruct the client that which of the following over-the-counter medications is contraindicated while taking oxycodone? A. Docusate sodium B. Ranitidine C. Diphenhydramine D. Ibuprofen

C. Diphenhydramine: Both diphenhydramine, an antihistamine, and oxycodone, an opioid analgesic, can cause CNS depression. Therefore, when a client uses the two medications together, the client is at increased risk for sedation, respiratory depression, and injury. Docusate sodium: Use of oxycodone can result in constipation. The client can take a stool softener, such as docusate, to manage this adverse effect. Ranitidine: There is no known interactions between oxycodone and rainitinde and no indication for the client to avoid the use of ranitidine while taking oxycodone. Ibuprofen: Clients who have musculoskeletal injury will benefit from using ibuprofen, and NSAID, in conjunction with an opioid analgesic.

A client is going to take ranitidine in the form of effervescent tablets. Which administration instructions for this drug formulation should be included? A. Swallow the tablets whole B. Chew the tablets before swallowing them C. Dissolve the tablets in water before drinking D. Place the tablets under the tongue

C. Dissolve the tablets in water before drinking The client should dissolve effervescent ranitidine tablets completely in 6 to 8 oz water and then drink the solution. The health care professional should make sure she understands that she should not chew them, swallow them whole, or dissolve them on or under the tongue. Additional instructions for clients taking this drug include taking the medication with or following meals and not smoking or drinking alcohol. (The Gastrointestinal System Case Study)

While talking with a client who is taking captopril to treat hypertension, which instructions do you include to help her manage the drugs adverse effects? A. Check your heart rate prior to taking the drug B. Wear sunscreen and protective clothing while outdoors C. Do not use salt substitutes to season your food D. Take the drug with a meal

C. Do not use salt substitutes to season your food Captopril, an angiotensin-converting enzyme inhibitor, can cause hyperkalemia due to potassium retention by the kidney, although it is rare unless the client is also taking potassium supplements, salt substitutes, or a diuretic that spares potassium. Therefore, caution the client about the use of salt substitutes, as most of them are high in potassium. The drug is not likely to alter heart rate, although it can cause palpitations. Although it is good advice for all clients to wear sunscreen and protective clothing during sun exposure, enalapril is not specifically known for causing photosensitivity. Administer captopril 1 hr before or 2 hr after meals for greatest absorption of the medication. (The Cardiovascular System Activity)

A nurse is collecting data from a client who has multiple sclerosis and a new prescription for baclofen. Which of the following findings should the nurse identify as an adverse effect of this medication? A. Diarrhea B. Weight loss C. Drowsiness D. Hypertension

C. Drowsiness: The nurse should identify drowsiness as an adverse effect of baclofen. Other adverse effects include dizziness, weakness, and fatigue. Diarrhea: The nurse should monitor for constipation as an adverse effect of baclofen. Weight loss: The nurse should monitor for weight gain as an adverse effect of baclofen. Hypertension: The nurse should monitor for hypotension as an adverse effect of baclofen.

A client is about to start taking oprelvekin for thrombocytopenia. To assess for adverse effects of this drug, you should monitor which of the following carefully during drug therapy? A. Liver function B. Hearing C. ECG D. Level of consciousness

C. ECG Oprelvekin, a thrombopoietic growth factor, can cause cardiac dysrhythmias, such as atrial fibrillation, atrial flutter, and tachycardia. Monitor this clients vital signs, heart rate, and ECG carefully. Instruct client to report palpitations, rapid pulse, or dizziness. Oprelvekin can cause peripheral edema and fluid retention, but it is unlikely to alter liver function. It is unlikely to affect level of consciousness, although it can cause insomnia and dizziness. (The Hematologic System Activity)

A nurse is reviewing the laboratory results of a client who takes insulin for the management of diabetes mellitus. Which of the following findings should indicate to the nurse the medication if effective? A. Fasting blood glucose 260 mg/dL B. HbA1c 9.2% C. Fasting blood glucose 100 mg/dL D. HbA1c 3%

C. Fasting blood glucose 100 mg/dL: The expected reference range for a fasting blood glucose level is 70 to 110 mg/dL. The nurse should identify that a client who has a fasting blood glucose level of 100 mg/dL is effectively managing diabetes mellitus. Fasting blood glucose 260 mg/dL: The expected reference range for a fasting blood glucose level is 70 to 110 mg/dL. The nurse should identify that a client who has a fasting blood glucose level of 260 mg/dL is hyperglycemic and does not indicate effective management of diabetes mellitus. HbA1c 9.2%: The expected reference range for HbA1c is 5.5% to 7%. The nurse should identify that a client who has HbA1c of 9.2% has poor diabetic control and does not indicate effective management of diabetes mellitus. HbA1c 3%: The expected reference range for HbA1c is 5.5% to 7%. The nurse should identify that a client who has HbA1c of 3% might have anemia, chronic blood loss, or chronic renal failure, which can result in low HbA1c results.

A client is about to start taking aliskiren to treat hypertension. Which of the following should you tell the client to avoid to ensure optimal absorption of this drug? A. Soy sauce B. Green, leafy vegetables C. Fatty foods D. Foods high in carbohydrates

C. Fatty foods Foods high in fat content reduce the absorption of aliskiren, a direct renin inhibitor. The client should not take this drug with a high-fat meal. Soy sauce, green leafy vegetables, and high-carbohydrate foods do not interfere with the absorption of this drug. Although low-sodium soy sauce is a better choice for this client, soy sauce can have serious adverse effects for clients taking monoamine oxidase inhibitors because it contains tyramine. Excessive amounts of green, leafy vegetables can interfere with the anticoagulant effects of warfarin, whose brand name is Coumadin. Foods high in carbohydrates can affect the overall glucose-controlling action of antidiabetic drugs. (The Cardiovascular System Activity)

You are caring for a client whose atrial dysrhythmia fails to respond to other antidsrhythmia therapies. You anticipate that the primary care provider might try which drugs whose off-label use includes refractory atrial dysrhythmias? A. Propranolol (Inderal) D. Lidocaine (Xylocaine) C. Flecaindine (Tambocor) D. Amiodarone (Cordarone)

C. Flecainide (Tambocor) Flecainide, a sodium channel blocker, treats serious ventricular dysrhythmias and also has an off-label use: atrial dysrhythmias other drugs have failed to control. Propranolol, a beta-adrenergic blocker, is especially useful for exercise- or stress-induced dysrhythmias. Lidocaine, also a sodium channel blocker, treats ventricular dysrhythmias. Amiodarone, a potassium channel blocker, treats recurrent ventricular fibrillation and ventricular tachycardia. (The Cardiovascular System Activity)

While obtaining a clients medical history, you find that the client is taking levodopa/carbidopa for Parkinson's disease. Which information in the client's health history alerts you to take further action? A. Bradycardia B. Pancreatic disease C. Glaucoma D. Impaired hearing

C. Glaucoma The combination of levodopa and carbidopa, both dopamine-replacement drugs, can cause changes in intraocular pressure. Primary care providers should not prescribe this drug combination for clients who have glaucoma, and in this case, the clients drug therapy requires adjustment. Pancreatic disorders are not contraindication for levodopa/carbidopa; instead, caution is required when using it with clients who have peptic ulcer disease. Hearing impairment is no contraindication for levodopa/carbidopa; instead, it can cause a variety of vison disturbance. (The Neurological System Activity (Part 1))

You are helping a client learn how to give himself an insulin injection. Which of the following is the best method for evaluating effective learning? A. verbally quiz a family member B. Instruct the client to complete a written test C. Have the client demonstrate an insulin injection D. Ask the client several key questions about the procedure

C. Have the client demonstrate an insulin injection The best way to evaluate a clients ability to perform a skill is through a return demonstration of that skill. Involving the family members and asking the client questions may help them retain the information and reinforce their learning, but they do not confirm the clients ability to perform the procedure. (Introduction to Pharmacology Activity)

A nurse is collecting data from a client who has angina and has a new prescription for sublingual nitroglycerin. Which of the following manifestations should the nurse expect as an adverse effect of this medication? A. Shortness of breath B. Bradycardia C. Headache D. Double vision

C. Headache: The nurse should expect the client to have headaches as a common adverse effect of taking sublingual nitroglycerin because it causes vasodilation. Shortness of breath: The nurse should not expect shortness of breath as an adverse effect of this medication. Nitroglycerin reduces oxygen demand and can improve breathing. Bradycardia: The nurse should expect the client to have tachycardia caused by sympathetic stimulation of the heart as an adverse effect of taking sublingual nitroglycerin. Double vision: The nurse should expect the client to have blurred visions as an adverse effect of taking sublingual nitroglycerin.

A nurse is assisting with the care of a client who has a methicillin-resistant staphylococcus aureus (MRSA) infection and is receiving vancomycin via IV infusion. Which of the following changes in the clients condition should the nurse identify as the priority finding to report to the provider? A. Nausea B. Back pain C. Hypotension D. Chills

C. Hypotension: When using the urgent vs. nonurgent approach to client care, the nurse determines that the priority finding to report to the provider is hypotension. If the clients vancomycin infusion is too rapid, it can cause red man syndrome, which is a group of adverse effects that includes tachycardia, hypotension, flushing, and urticaria Nausea: Although the nurse should report nausea for a client who is receiving vancomycin therapy, there is another finding that the nurse priority to report. Back pain: Although the nurse should report back pain for a client who is receiving vacomycin therapy, there is another finding that the nurses priority to report. chills: Although the nurse should report chills for a client who is receiving vancomycin therapy, there is another finding that is the nurses priority to report.

A nurse is caring for a client who has a 10-year history of alcohol use disorder and is experiencing acute alcohol withdrawal. The nurse should identify which of the following interventions as the priority? A. Suggest the client attend a support group. B. Administer naltrexone. C. Implement seizure precautions. D. Assist the client to identify triggers of alcohol use.

C. Implement seizure precautions.: The greatest risk to the client is injury from seizures and falls. Grand mal seizures can occur during severe alcohol withdrawal. Therefore, the nurses priority is to implement seizure precautions to prevent injury if the client experiences a seizure. Suggest the client attend a support group.: The nurse should suggest the client attend a support group to provide ongoing support assistance with maintaining abstinence after detoxification. However, another intervention is the nurses priority. Administer naltrexone.: The nurse should administer naltrexone to the client to assist with maintaining abstinence after detoxification. However, another intervention is the nurses priority. Assist the client to identify triggers of alcohol use.: The nurse should assist the client with identifying triggers of alcohol use to assist with maintaining abstinence after detoxification. However, another intervention is the nurses priority.

You are about to administer diphenoxylate plus atropine to a client to reduce diarrhea. Which drugs should you have available to treat an overdose of this drug combination? A. Acetylcysteine B. Flumazenil C. Naloxone D. Diphenhydramine

C. Naloxone With high doses, diphenoxylate plus atrophine, an opioid agonist antidiarrheal drug combination, can cause CNS effects similar to morphine. Have naloxone ready in case of drug overdose. Monitor respirations and level of sedation. Flumazenil reverses the effects of benzodiazepines. Diphenhydramine reduces extrapyramidal symptoms resulting from metoclopramide use. (The Gastrointestinal System Activity)

A nurse is reinforcing teaching with a client who is newly diagnosed with hypertension and is taking metoprolol. The nurse should instruct the client to report which of the following manifestation to the provider as an adverse effect of this medication? A. Tachycardia B. Tinnitus C. Peripheral edema D. Urinary retention

C. Peripheral edema: The nurse should instruct the client to monitor for and report the development of peripheral edema because this can be an indication of heart failure, which is an adverse effect of metoprolol. Tachycardia: The nurse should instruct the client to monitor for bradycardia, rather than tachycardia. Tinnitus: The nurse should instruct the client to monitor for blurred vision rather than tinnitus. Urinary retention: The nurse should instruct the client to monitor for urinary frequency, rather than urinary retention.

A client receives spinal anesthesia with lidocaine and epinephrine. You explain that epinephrine is used with lidocaine for which of the following reasons? A. Decreases the risk for a spinal headache B. Reduces the risk for infection C. Prolongs the anesthetic effects D. Prevents respiratory depression

C. Prolongs the anesthetic effects Injecting epinephrine with lidocaine for local or regional anesthesia extends the duration of the regional anesthesia. Epinephrine will not reduce the risk for a spinal headache. Instead, positioning clients supine can reduce headaches. Epinephrine has no effect on infection risk. It is essential for the anesthetist to confirm correct drug concentration and correct spinal placement to prevent drug toxicity and to monitor for paresthesia, confusion, and agitation, which can lead to seizures and respiratory arrest. Injecting epinephrine into the subarachnoid space will not prevent respiratory depression. A spinal anesthetic that involves the intercostal muscles can cause respiratory depression. Should this occur, it is crucial to monitor the spinal level, administer supplemental oxygen, and provide respiratory support as needed. (The Neurological System Activity (Part 1))

Which of the following drugs should the nurse have available to reverse the adverse effects of terbutaline? A. Naloxone B. Neostigmine (Prostigmin) C. Propranolol (Inderal) D. Atropine

C. Propranolol (Inderal) Propranolol is a beta-adrenergic antagonist. It slows the heart rate and reduces the cardiac irritability terbutaline causes. Naloxone is an opioid antagonist that reverses the effects of opioids. Neostigmine, a cholinesterase inhibitor, reverses the effects of pancuronium, a nondepolarizing neuromuscular blocking agent. Atropine is an anticholinergic drug that reverses the effects of acetylcholine or muscarinic drugs. (The Reproductive and Genitourinary Systems Case Study)

A nurse is monitoring a client who has just returned to the unit following surgery. The client received an initial dose of morphine 5 mg IV bolus for pain relief. Which of the following adverse effects should the nurse report immediately to the provider? A. Emesis B. Sedation C. Respiratory rate 11/min D. Blood pressure 100/74 mmHg

C. Respiratory rate 11/min: When using the airway, breathing, circulation approach to client care, the priority finding is a respiratory rate of 11/min, which indicates respiratory depression. Emesis: The nurse should administer an antiemetic to treat emesis, which can be an adverse effect of opioids. However, another finding is the priority. Sedation: The nurse should implement fall precautions for sedation, which can be an adverse effect of opioids. However, another finding is the priority. Blood pressure 100/74 mmHg: This blood pressure is within the expected reference range. The nurse should request a fluid bolus if the client experiences low blood pressure, which can be an adverse effect of opioids. However, another finding is the priority.

A nurse is collecting data from a client who is taking lithium to treat bipolar disorder. Which of the following findings should the nurse report to the provider? A. Bloating B. WBC count 8,500/mm^3 C. Slurred speech D. Sodium 140 mEq/L

C. Slurred speech: The nurse should recognize that slurred speech is an indication of toxicity and should be reported to the provider. Bloating: The nurse should recognize bloating is a transient adverse effect of lithium and does not need to be reported to the provider. WBC count 8,500mm^3: The nurse should recognize that a WBC count of 8,500mm^3 is within the expected reference range and does not need to be reported to the provider. Lithium can cause leukocytosis. Sodium 140 mEq/L: the nurse should recognize that a sodium level of 140 mEq/L is within the expected reference range and does not need to be reported the provider.

A nurse is monitoring a client who is receiving a transfusion of packed RBCs. The clients temperature increases 39.1 C(102.4 F). Which of the following actions should the nurse take first? A. Obtain a urine specimen. B. Administer diphenhydramine. C. Stop the transfusion. D. Notify the charge nurse.

C. Stop the transfusion: The greatest risk to this client is injury from an elevated temperature; therefore, the first action the nurse should take is to stop the transfusion. Obtain a urine specimen: The nurse should obtain a urine specimen to determine the extent of the reaction; however, this is not the first action the nurse should take. Administer diphenhydramine: The nurse might need to administer an antihistamine, such as diphenhydramine, as an emergency medication; however, this is not the first action the nurse should take. Notify the charge nurse: The nurse should notify the charge nurse of possible transfusion reaction because it is an emergent situation; however, this is not the first action the nurse should take.

A client is given conjugated estrogen and medroxyprogesterone acetate to treat post menopausal vasomotor symptoms. The health care provider instructs the client to monitor for which of the following, which may indicate a serious adverse effect of the conjugated estrogen and medroxyprogesterone acetate? A. Ringing in ears B. Insomnia C. Swelling or pain in calf D. Difficulty swallowing

C. Swelling or pain in calf Conjugated estrogen and medroxyprogesterone acetate, a combination estrogen plus progestin hormone replacement, may cause thromboembolism. The client should be instructed to report any redness or swelling of the calf or shortness of breath. Conjugated estrogen and medroxyprogesterone acetate is unlikely to cause tinnitus. Conjugated estrogen and medroxyprogesterone acetate may cause nausea, vomiting and constipation. It is unlikely to cause insomnia. Alendronate may cause difficulty swallowing. (The Reproductive and Genitourinary Systems Activity)

The provider should instruct the client to do which of the following? (One month after taking clomiphene the client has not started her monthly period.) A. Take half the usual dosage B. Continue the usual dosage C. Take a pregnancy test D. Double the previous dosage

C. Take a pregnancy test Clomiphene is pregnancy risk with teratogenic effects. If there is any suspicion of pregnancy, the client should stop taking the drug immediately and take a pregnancy test. (The Reproductive and Genitourinary Systems Case Study)

A primary care provider prescribes etanercept to treat a clients rheumatoid arthritis. Prior to beginning the treatment, the client requires testing for which of the following? A. Pancreatitis B. Hepatitis C. Tuberculosis D. Gastritis

C. Tuberculosis Primary care providers should test the clients for tuberculosis prior to treatment with etanercept. Pulmonary tuberculosis in clients taking etanercept can spread to other organs and pose treatment challenges. They should also monitor clients for the development of tuberculosis during etanercept therapy. (The Musculoskeletal System Activity)

A client is prescribed testosterone to treat testicular failure. The nurse instructs the clients to monitor and report which of the following? A. Dizziness B. Paresthesia C. Weight gain D. Shortness of breath

C. Weight gain Instruct the client to monitor weight. Testosterone may cause sodium and water retention and edema. Monitor intake and output and weight and sodium level. Instruct client to report swelling in legs or feet. Testosterone is unlikely to cause paresthesia or dizziness. Clomiphene may cause dizziness. Testosterone is unlikely to cause shortness of breath. Shortness of breathe may be caused by thromboembolism, an adverse effect of medroxyprogesterone acetate. (The Reproductive and Genitourinary Systems Activity)

You are talking with a client about taking tetracycline along with antacid. You tell the client not to take these two drugs at the same time because the antacid can reduce the absorption of tetracycline. When one drug reduces the effect of another drug, it is A. an allergic reaction B. a synergistic effect C. an antagonistic effect D. an adverse reaction

C. an antagonistic effect When one drug reduces the effect of another drug, it is an antagonistic effect. When the clients body responds to a drug by releasing histamine and causing itching and hives, the client is having an allergic reaction to a drug. Drugs that interact, causing greatly increased or exaggerated effects, are acting synergistically. The nontherapeutic, unintended effects of a drug that occur at a therapeutic dose or side or adverse effects or reactions. (Introduction to Pharmacology Activity)

A nurse is contributing to the plan of care for a client who has schizophrenia and a new prescription for clozapine. The nurse should include in the plan to monitor the client for which of the following adverse effects of this medication? A. Hypoglycemia B. Iron-deficiency anemia C. Serotonin syndrome D. Agranulocytosis

D. Agranulocytosis: The nurse should monitor the clients WBW count and notify the provider for a value below the expected reference range of 5,000 to 10,000 mm^3. Hypoglycemia: The nurse should monitor the client for hyperglycemia, rather than hypoglycemia. Iron-deficiency anemia: The nurse should monitor the client for hyperlipidemia, rather than iron-deficiency anemia. Serotonin syndrome: The nurse should monitor the client for tardive dyskinesia, rather than serotonin syndrome.

A nurse is reinforcing teaching with a client who has Helicobacter pylori and a new prescription for tetracyline. Which of the following instructions should the nurse include in the teaching? A. "Expect your urine to turn orange." B. "Take the medication with 240 milliliters (8 ounces) milk." C. "Watch for excessive bleeding when brushing your teeth." .D. "Avoid prolonged exposure to sunlight."

D. "Avoid prolonged exposure to sunlight.": The nurse should instruct the client to avoid prolonged exposure to sunlight while taking tetracycline. This medication causes photosensitivity and increased severity of sunburn. "Expect your urine to turn orange.": Tetracycline does not cause changes in urine color. However, it can affect kidney functions test, such as BUN and creatinine. "Take the medication with 240 milliliters (8 ounces) milk.": The client should avoid taking tetracycline with foods high in calcium, such as milk. This decreases the absorption of the medication. "Watch for excessive bleeding when brushing your teeth.": Tetracycline does not cause bleeding complications. However, it can cause fungal infections, such as candidiasis.

A nurse is reinforcing teaching with a client who has new prescription for fluticasone inhaler. Which of the following client statements indicates on understanding of the teaching? A. "I will administer two puffs of the medication consecutively." B. "I will use this inhaler if I feel an asthma attack coming on." C. "I will avoid intake of diary products in my diet." D. "I will rinse my mouth after I will this inhaler."

D. "I will rinse my mouth after I will this inhaler.": The client should rinse his mouth after using inhaled glucocorticoids to avoid the development of oral thrush. Therefore, the nurse should identify this statement as indicating an understanding of the teaching. "I will administer two puffs of the medication consecutively.": The nurse should instruct the client to wait 1 to 2 min between inhalations of the medication to increase absorption. "I will use this inhaler if I feel an asthma attack coming on.": the nurse should instruct the client to use this medication as prophylactic measure to prevent an asthma attack. "I will avoid intake of diary products in my diet.": The nurse should instruct the client to increase intake of dairy products to reduce the risk for bone loss while taking this medication.

A nurse is reinforcing teaching with a client who has prescription for alendronate. Which of the responses indicate to the nurse an understanding of teaching? A. "I will take the medication with my breakfast." B. "I will take the medication with 1 tablespoon of an antacid." C. "I will lie down for 30 minutes after taking the medication." D. "I will take the medication with 8 ounces of water."

D. "I will take the medication with 8 ounces of water.": The client should take alendronate on an empty stomach with 240 mL (8 oz) of water to ensure it does not lodge in the esophagus, which can result in esophageal ulcerations. "I will take the medication with my breakfast.": The client should take alendronate upon arising and at least 30 minutes before eating or drinking liquids other than water. "I will take the medication with 1 tablespoon of an antacid.": The client should avoid taking alendronate with antacids containing calcium because they can decrease absorption of alendronate. "I will lie down for 30 minutes after taking the medication.": The client should sit upright for 30 min after taking the medication because alendronate can cause erosion of the esophagus.

A nurse is preparing to administer a PRN medication to a group of clients . Which of the following clients should the nurse administer medication to first? A. A client who has GERD and requests an antacid B. A client who reports constipation for 3 days and requests a stool softener C. A client who has mild generalized anxiety disorder and requests an antianxiety medication D. A client who is attending postoperative physical therapy and requests pain medication

D. A client who is attending postoperative physical therapy and requests pain medication: When using the urgent vs nonurgent approach to client care, the nurse should determine the first client to medicate is the client who is postoperative and going to physical therapy. The client can experience pain during and after therapy. A client who has GERD and requests an antacid: Requesting an antacid is nonurgent because heartburn is an expected finding for a client who has GERD. Therefore, there is another client the nurse should administer medication to first. A client who reports constipation for 3 days and requests a stool softener: The client who reports constipation and requests a stool softener is nonurgent. Therefore, there is another client the nurse should administer medication to first. A client who has mild generalized anxiety disorder and requests an antianxiety medication: Requesting an antianxiety medication is nonurgent for a client who has mild generalized anxiety disorder. Therefore, there is another client the nurse should administer medication to first.

You are instructing a client who has a new prescription for cetirizine. Which of the following information should you be sure to include? A. Restrict fluids to minimize the drugs adverse effects B. Take the drug in the morning to reduce adverse effects C. Take the drug on an empty stomach to increase absorption D. Administer the drug prophylactically during allergy seasons

D. Administer the drug prophylactically during allergy seasons Cetirizine is a second-generation antihistamine used primarily to relieve symptoms associated with seasonal allergic rhinitis. Increasing fluids if possible may help reduce dry mouth and constipation. Taking cetirizine in the morning will not minimize its adverse effects. Taking antihistamines with food may help reduce gastric distress. (The Respiratory System Activity)

A nurse is preparing to administer diphenhydramine 50 mg PO at 2200 to a client who has difficulty swallowing pills and capsules. Available is diphenhydramine syrup 12.5 mg/5 mL PO. Which of the following nursing actions requires the completion of an incident report? A. Giving the medication at 2140 B. Administering the medication with grapefruit juice C. Giving the medication when the clients apical pulse is 58/min D. Administering 25 mL of the syrup

D. Administering 25 mL of the syrup: This dose is higher than the client should receive. The correct dosage is 20 mL. Administering an incorrect amount of medication to a client requires a completion of an incident report. Giving the medication at 2140: Administering the medication at 2140 is within the appropriate time frame. Administering the medication with grapefruit juice: Although grapefruit juice can alter the action of a number of medications, it does not affect diphenhydramine. Giving the medication when the clients apical pulse is 58/min: The nurse can administer diphenhydramine if the clients heart rate is below 60/min because this medication can increase the heart rate.

You are instructing a client who has multiple sclerosis about how dantrolene will help control spasticity. With which mechanism is this drug effective in treating this manifestation? A. Blocks acetylcholine at the neuromuscular junction B. Interrupts nerve signals from the spinal cord to the muscles C. Inhibits destruction of acetylcholine by acetylcholinesterase D. Alters intercellular calcium exchange at the muscles

D. Alters intercellular calcium exchange at the muscles Dantrolene, a peripherally acting muscle relaxant, acts directly on spastic muscles and inhibits muscle contraction by preventing the release of calcium in skeletal muscles. Anticholinergic drugs or muscarinic antagonists block acetylcholine at the neuromuscular junction. Baclofen, a centrally acting muscle relaxant, relieves spasticity by interrupting nerve signals from the spinal cord. .Most drugs used to treat myasthenia gravis act by preventing the destruction of acetylcholine at the neuromuscular junction. (The Neurological System Activity (Part 1))

A nurse is collecting data prior to administering digoxin to a client. For which of the following findings should the nurse withhold this medication and notify the provider? A. Digoxin level 0.9 ng/mL B. Blood pressure 142/80 mmHg C. Potassium 4.4 mEq/L D. Apical pulse 52/min

D. Apical pulse 52/min: The nurse should check the clients apical pulse prior to administering digoxin because it decreases the heart rate and improves contractility. If the clients heart rate is below 60/min, the nurse should withhold the dose and notify the provider. Digoxin level 0.9 ng/mL: The clients digoxin level is within the expected reference range. Therefore, the nurse should not withhold the digoxin in response to this finding. Blood pressure 142/80 mmHg: Digoxin has hypotensive effects. The nurse should identify this blood pressure reading as hypertension, which does not require withholding the digoxin. Potassium 4.4 mEq/L: Hypokalemia places clients at risk for cardiac dysrhythmias when taking digoxin. However, the clients potassium level is within the expected reference range and does not require the nurse to withhold the digoxin.

A nurse is monitoring a client who is 2 hr postoperative and has a prescription for opioid analgesics. Which of the following actins provides the nurse with the priority data to determine the clients need for analgesia? A. Observe the client for signs of restlessness. B. Monitor the client for facial grimacing. C. Watch the client for indications of decreased mobility. D. Ask the client to rate her pain level.

D. Ask the client to rate her pain level.: According to evidence-based practice, the nurse should first ask the client to rate her pain level to provide a verbal report of pain, which is the priority indicator of the need for pain medication. Observe the client for signs of restlessness.: The nurse should monitor the client for restlessness as an indication of pain. However, evidence-based practice indicates that another action is the priority. Monitor the client for facial grimacing.: The nurse should monitor the client for facial grimacing as an indication of pain. However, evidence-based practice indicates that another action is the priority. Watch the client for indications of decreased mobility.: The nurse should monitor the client for decreased mobility as an indication of pain. However, evidence-based practice indicates that another action is the priority.

A nurse is performing the third check before administering hydromorphone to a client. After opening the unit-dose packet, the client tells the nurse he does not want to take the medication now. Which of the following actions should the nurse take? A. Complete an occurrence report of the incident. B. Encourage the client to take the medication. C. Leave the medication at the clients bedside in case he changes his mind. D. Dispose of the medication with a second nurse as a witness.

D. Dispose of the medication with a second nurse as a witness.: The nurse is legally required to have witness when disposing of a controlled substance. Complete an occurrence report of the incident.: The nurse does not need to complete an occurrence report when discarding a controlled substance. However, the nurse should record the disposal of the medication. Encourage the client to take the medication.: The nurse should respect the clients right to refuse the medication. Leave the medication at the clients bedside in case he changes his mind.: The nurse should not leave the medication at the clients bedside because hydromorphone is controlled substance that should not be left unattended.

A nurse erroneously administered zolpidem to the wrong client. Which of the following actions should the nurse take when documenting the medication error? A. Document the completion of the incident report in the medical record of the client who received the zolpidem. B. Administer a dose of naloxone to reverse the effects of the medication. C. Keep a copy of the incident report on the unit. D. Document the notification of the clients provider.

D. Document the notification of the clients provider.: In the medical record of the client who received the zolpidem, the nurse should document the objective facts of the error, including follow-up actions and notification of the provider. Document the completion of the incident report in the medical record of the client who received the zolpidem.: Practice errors can lead to litigation. A notation on the medical record about the completion of an incident report can lead to a prosecuting attorney gaining access to it because it is part of the clients medical record. Administer a dose of naloxone to reverse the effects of the medication.: The nurse should notify the provider and plan to do additional monitoring. Naloxone is used to reverse the effects of opioids. Keep a copy of the incident report on the unit.: Incident reports are confidential. The nurse should follow the facility's protocol for handing these reports, which usually includes submitting the report to the risk management department and not keeping a copy where others might access it.

The health care provider prescribes alendronate (Fosamax) to reduce bone resorption and instructs the client to monitor for which of the following indications of a serious adverse effect of this drug? A. Calf inflammation B. Chest pain C. Dizziness D. Dysphagia

D. Dysphagia Dysphagia (difficulty swallowing) can indicate esophagitis, a rare but serious effect of alendronate. The client must understand that she has to notify the provider of any pain or difficulty swallowing and of worsening heartburn. Alendronate is unlikely to cause deep vein thrombosis. This serious adverse effect is more likely with raloxifene (Evista). Alendronate is more likely to cause musculoskeletal pain than chest pain. Alendronate is unlikely to cause dizziness. Dizziness might accompany bradycardia, a potentially serious adverse effect of neostigmine (Prostigmin) and of parenteral calcium interaction with digoxin (Lanoxin.) (The Musculoskeletal System Case Study)

A nurse is evaluating a client who is receiving amphotericin B via intermittent IV bolus. Which of the following findings indicates an adverse reaction to this medication? A. Serum potassium 5.6 mEq/L B. Hematocrit 55% C. Polyuria D. Hypotension

D. Hypotension: The nurse should identify that amphotericin B is considered a high-alert drug due to potentially serious adverse effects, such as hypotension. Therefore, the nurse should report this or other adverse effects of amphotericin, such as nephrotoxicity, hypokalemia, and cardiac dysrhythmias. Serum potassium 5.6 mEq/L: The nurse should monitor the client for a decreased potassium level because amphotericin B can cause hypokalemia. However, a serum potassium of 5.6 mEq/L is above the expected reference range, rather than below. Although this is not an adverse effect of amphotericin B, the nurse should report it the provider. Hematocrit 55%: The nurse should monitor the client for a decreased hematocrit level because amphotericin B can cause bone marrow suppression, resulting in anemia. However, a hematocrit of 55% is above the expected reference range, rather than below. Although this is not an adverse effect of amphotericin B, the nurse should report it the provider. Polyuria: The nurse should monitor the client for decreased urinary output because amphotericin B can cause nephrotoxicity, leading to damage of the kidneys.

Which of the following instructions should the health care professional give a client about taking sucralfate to treat an acute duodenal ulcer? A. Take it with a prescribed antacid B. Reduce potassium intake C. Take it with food D. Increase fluid and fiber intake

D. Increase fluid and fiber intake Sucralfate, a mucosal protectant, can cause constipation, so the client should increase intake of fluids and fiber. She should not take antacids within 30 -60 minutes of taking sucralfate. The client should take sucralfate on an empty stomach to increase absorption. Sucralfate can be taken without regard to potassium intake. (The Gastrointestinal System Case Study)

A nurse is reinforcing teaching with a client who has rheumatoid arthritis and a new prescription for methotrexate. Which of the following information should the nurse include in the teaching? A. Avoid grapefruit juice. B. Increase salt intake. C. Avoid aged cheese. D. Increased fluid intake.

D. Increased fluid intake.: Client who are taking methotrexate should increase fluid intake to reduce the risk for renal damage and to increase drug excretion. Avoid grapefruit juice.: Grapefruit juice is not contraindicated for clients taking methotrexate. Increases salt intake.: Clients who are taking methotrexate do not need to increase salt intake. Avoid aged cheese.: Aged cheeses are not contraindicated for clients taking methotrexate.

A nurse is collecting data from a client who is taking ferrous sulfate orally. Which of the following findings reported by the client should indicate to the nurse that the medication is having a therapeutic effect? A. Passage of a soft, formed stool daily B. Decreased number of viral illnesses C. Improved ability to fall asleep D. Increased tolerance to exercise

D. Increased tolerance to exercise: The client who takes ferrous sulfate, which is used to treat iron-deficiency anemia. Can have fatigue and shortness of breath due to a low hemoglobin level. An increased tolerance to exercise is an indication the ferrous sulfate is having a therapeutic effect. Increased tolerance to exercise occurs when the hemoglobin level increases, allowing more oxygen to be carried to the vital organs and tissue. Passage of a soft, formed stool daily: Passing a soft, formed stool is not an indication the medication is having therapeutic effect. Ferrous sulfate can cause constipation. Decreased number of viral illnesses: A decreased number of viral illnesses is not indication the ferrous sulfate is having a therapeutic effect. Improved ability to fall asleep: An improved ability to fall asleep is not an indication the ferrous sulfate is having a therapeutic effect.

A nurse is reinforcing teaching with a client who has bipolar disorder and a new prescription for lithium. Which of the following instructions should the nurse include in the teaching. A. Take the medication on an empty stomach. B. Monitor for signs of hyper hyperthyroidism. C. Watch for sings of urinary retention. D. Maintain a consistent sodium intake.

D. Maintain a consistent sodium intake. Take the medication on an empty stomach.: The client should take lithium with meals or milk to reduce gastrointestinal upset. Monitor for signs of hyper hyperthyroidism.: The nurse should instruct the client that hypothyroidism can occur when taking lithium. Watch for sings of urinary retention.: The nurse should instruct the client that polyuria can occur when taking lithium.

You are caring for a client who is taking exenatide (Byetta) to treat type 2 diabetes mellitus. The client reports severe abdominal pain. You suspect which of the following adverse reactions to this drug. A. Peptic ulcer disease B. Hyperkalemia C. Hyperglycemia D. Pancreatitis

D. Pancreatitis Exenatide, an incretin mimetic agent, can cause acute pancreatitis. You instructed the client to watch for and report severe or persistent abdominal pain, sometimes radiating to the back, may or may not be accompanied by vomiting at beginning of therapy and with dose increases, so you should now inform the primary care provider and tell the client to stop taking the drug. Exenatide is unlikely to cause hyperkalemia, an effect much more likely with insulin. For clients taking insulin, monitor potassium levels and instruct them to report muscle weakness or palpitations. Exenatide is more likely to cause hypoglycemia than hyperglycemia, so monitor blood glucose levels carefully and administer a carbohydrate for hypoglycemia. Indications of hypoglycemia include tachycardia, diaphoresis, shakiness, and weakness. (The Endocrine System Activity)

A client starts drug therapy, but the following day he reports that he is having palpitations. His ECG at the emergency department indicates frequent premature ventricular contractions, so the primary care provider prescribes a stat dose of lidocaine, 50 mg via IV bolus. A health care profession who administers the drug monitors the client for which adverse reactions to lidocaine? A. Metabolic alkalosis B. Hypertension C. Tachycardia D. Paresthesias

D. Paresthesias High therapeutic doses of lidocaine, a class 1B antidysrhythmic, can cause paresthesias, such as numbness and tingling, which can lead to seizure activity with repeated doses. This drug is unlikely to cause metabolic alkalosis, but high doses can cause hypotension and bradycardia. (The Cardiovascular System Case Study)

Which of the following assessments of this client is most important immediately following the administration of intravenous diazepam? A. ECG B. Seizure activity C. Blood pressure D. Respiratory rate

D. Respiratory rate It is important to monitor the clients cardiac rhythm following the administration of IV diazepam, a benzodiazepine, due to the risk of cardiac arrest. However, this is not the most important assessment. It is also important to monitor seizure activity to determine the effectiveness of the diazepam, a benzodiazepine, but this is not the most important assessment. It is also important to monitor all vital signs following the administration of IV diazepam, a benzodiazepine, due to the risk of hypotension and cardiac arrest. Nonetheless, this is not the most important assessment. The greatest risk to a client at this time is respiratory depression and apnea. Therefore, it is most important for the health care profession to assess the clients respiratory rate. IN addition, the client may need supplemental oxygen. (The Neurological System Case Study (Part 1))

A client is about to start therapy with methotrexate for rheumatoid arthritis. Knowing the adverse effects of methotrexate, you advise the client to watch for which of the following? A. Dysphagia B. Sore throat C. Edema D. Parenthesis

D. Sore throat Methotrexate may cause bone marrow suppression. The client should report any sign of infection, such as sore throat or fever. Dysphagia, edema, and parenthesis (numbness or tingling sensations on the skin) are not common adverse effects of methotrexate. Adverse effects of this drug include headache, mucositis, gastric ulcers, gingivitis, and hepatotoxicity. (The Musculoskeletal System Activity)

Which of following information indicates that a client who is taking theophylline needs further instruction? A. The client drives a school bus every day B. The client drinks 8 oz.. of wine with dinner C. The client exercises 1 hour each day D. The client drinks two cups of coffee each morning

D. The client drinks two cups of coffee each morning Caffeine may increase CNS stimulations, causing nervousness, insomnia, and tremors. It may also increase cardiac simulation and cause tachycardia. Instruct clients taking methylxanthines to avoid caffeine intake. Alcohol use is not a contraindication for theophylline use. Exercise is appropriate for clients who have asthma. Theophylline should not impair the clients ability to operate a motor vehicle. Primary care providers should no prescribe theophylline for clients who have severe liver or renal impairment. (The Respiratory System Activity)

A nurse is preparing to administer phenobarbital 3mg/kg PO twice a day to a school-age child who weighs 44 lb. Available is phenobarbital elixir 20 mg/5 mL. How many mL should the nurse plan to administer per dose? (Round the answer to the nearest whole number. Use a leading zero if it applies. Do not use a trailing zero.) Answer is fill in the blank.

Measurements the nurse should calculate? kg 44 lb / 2.2 = kg: 20 mg x kg/dose = X 3 mg x 20 kg = 60 mg/dose Measurements the nurse should calculate to administer? kg Dose to administer = Desired 60 mg Dose available = 20 mg Dose quantity = 5 mL 60 mg x 5 mL/20 mg = X mL X = 15 mL


Ensembles d'études connexes

Module 9 (ch.13&14) Sourcing and Logistics

View Set

Theatre:A Lively Art 8th Edition HigherEd Quiz

View Set

CISM 2201 Mod. 3: Computer Hardware

View Set

Chapter 5 Assess What You Learned

View Set

ARDMS Ultrasound Physics & Instrumentation Practice Exam

View Set